Você está na página 1de 57

Torts Professor Nockleby

Multiple Choice Practice Questions—Intentional Torts & Defenses

The following practice questions have been culled from bar exam preparation materials written by BAR-BRI and
PMBR. (These are courses offered by private companies that most students preparing for the California Bar exam
will take following law school.) Many MC practice books are held on reserve in the Library.

An Answer Key to the following questions may be found towards the end of the packet. Since there are only 70
questions, you should use these questions to test yourself under timed conditions. Do not assume that these
questions reflect all types of questions that you may be asked during your first year torts exam, but they illustrate
at least some types of questions you are likely to see. Also, since Multistate is a national test, some of the answers
in this packet presuppose knowledge of torts or defenses you’ve not studied (e.g., privacy). Please note that your
torts multiple choice exam will not test you on any torts or defenses you have not studied.

***
For Questions 1 – 3:

In order to demonstrate the inadequacy of most witnesses’ perceptions to her trial practice class, Professor
Seapnor arranged a phony attack scene with one of the students, Martin. Martin was to enter the classroom a few
minutes late, yell some obscenity at Professor Seapnor regarding her grading policies, and then charge at her,
seeming to be attempting to choke her. Professor Seapnor would then appear to subdue him with a martial arts
hold. The scene was to end with the student felling and the professor directing the class to write one paragraph
describing the details of the incident. Professor Seapnor had invited her husband, Johnson, also a professor, to join
the class that evening to assess the effectiveness of the technique.

All went as planned during the scene until Professor Seapnor was “subduing” Martin with the martial arts
hold. At that moment, Finn, a law student and police officer, entered the classroom, and believing that Professor
Seapnor had finally gone berserk, yelled a warning and drew his service revolver. To evade the shot, Professor
Seapnor dove to the floor behind a table, knocking Belinda from her chair in such a fashion that Belinda was hit
by Finn’s single shot.

Martin saw the classroom panic as the perfect opportunity to get even with Professor Seapnor for giving
him the lowest grade he had ever received in law school, an 84 in Torts. Martin drew a switchblade from his
pocket and stabbed Professor Seapnor several times in the ribs. The only person to observe Martin’s act was
Johnson.

Professor Seapnor recovered, although she was reprimanded by the Dean for staging such an incident in light
of the fact that she knew numerous students in each class to be police officers, private investigators, or others
carrying licensed weapons. Johnson has recurring nightmares of the incident and required several months of
therapy before he felt comfortable returning to teaching.

1. If Belinda sues Finn for battery, will she prevail?

(A) No, because Finn did not know to a substantial certainty that a touching would occur.

(B) No, because Finn reacted reasonably to the danger apparently posed to Martin.

(C) Yes, because Finn intended to place Seapnor in apprehension of a touching.


Finn had the intention to place seapnor in an apprehension with his shot gun and when he shot the gun
aiming for seapnor, but missed, his intent was transferred to Belinda.

450121682.doc Page 1 of 57
(D) Yes, because a reasonable police officer would not have fired his revolver in a classroom setting.

2. If Belinda sues Seapnor, what is the probable outcome?

(A) Seapnor will prevail because she was attempting to protect herself from Finn’s shot.

(B) Belinda will prevail, but only if Seapnor was negligent in evading Finn’s shot.
= it is not only if she was negligent in evading flinn’s shot. Remember that seapnor is a teacher. So she had a duty
as a teacher. The fact pattern also states that she KNEW students in the classroom carried licensed guns so she
could have also been negligent in the set up of her lesson.

(C) Seapnor will prevail because Finn was an intervening tortious force.

(D) Belinda will prevail, because Seapnor knew students in the classroom carried licensed guns.

3. If Johnson sues Martin on a theory of intentional infliction of emotional distress, what must plaintiff
prove to recover damages?

I. Martin intended to injure Johnson. = intent to injury not required. Only intent to do the
extreme and outrageous conduct. The actor intended the place P in ED or KWSC that he
would be.

II. Martin’s behavior was extreme and outrageous.

III. Johnson suffered physical harm. = physical not required

IV. Johnson suffered severe emotional distress.

(A) II and III.

(B) II and IV.

(C) I and IV.

(D) I and III.

4. Gilda Gammary, a precocious student at Northcentral High School, was an “A” student in her chemistry
class and was interested in developing a Science Fair project in the area of chemistry. She was inspired by
an experiment conducted in class in which the teacher had the students mix three chemicals together to
create a gas that caused the faces of the students to become grossly distorted. Gilda obtained an ample
supply of the three chemicals and went to an abandoned building located on a street that had heavy
pedestrian traffic. She mixed together the chemicals, and the fumes passed across the sidewalk causing
the pedestrians’ faces to become grossly distorted, as if they suffered from physical defects. The effect of
the gas was temporary and none of the pedestrians suffered any permanent damage. One of the
pedestrians exposed to the gas was Parker.

If Parker wants to sue Gilda, which of the following best describes the tort she has committed against
him?

450121682.doc Page 2 of 57
(A) Assault. = only if pedestrians were aware

(B) Battery. = smoke can constitute as offensive tuching = exposed to gas.

(C) Intentional infliction of emotional distress. = has to state that parker suffered from emotional distress.
Don’t just make it up that it exists

(D) Invasion of privacy. = on a public street

5. Trucko was driving a dumptruck containing gravel at a safe speed. From the other direction Crasher drove
his big semitrailer truck. Crasher came around a curve and was confronted with a very slow- moving
steamroller just in front of him. To avoid colliding with the steamroller, Crasher pulled his wheel to the
left and crossed the center lane, where he bore down on Trucko, who was coming from the other
direction. Crasher showed no propensity to yield, and there were other vehicles to Trucko’s left. Trucko’s
only option was to turn his truck to the right, onto the land of Paget. The truck caused damage to Paget’s
lawn.

Which of the following best describes Trucko’s liability to Paget?

(A) Tucko is liable for the damage to the lawn.

(B) Trucko is liable for public nuisance, because he has interfered with Paget’s use and enjoyment of
land. = not long enough

(C) Tucko is liable for both trespass and public nuisance. = not liable for nuisance needs to state that

(D) Trucko is liable for nothing, because the incident was not his fault, and he acted in a reasonable and
responsible manner. = private necessity- you are still liable for damages

6. Porphyry and her friend Martha entered “Maison Mamselle,” a women’s clothing store. As the store was
rather warm, both women removed their coats and carried them over their arms as they shopped.
Porphyry’s coat was a brown suede coat, which she had purchased on a recent trip to Italy. Suzie Sales, a
clerk in the store, saw Porphyry and Martha looking at the rack where Maison Mamselle keeps its suede
coats. Ten minutes later she saw Martha help Porphyry put on a brown suede coat, which happened to be
the coat Porphyry bought in Italy, although Suzie did not know that. When Suzie saw Porphyry leaving
the store wearing the brown suede coat, she immediately notified Dagobert, the store detective, telling
him, “That woman stole that brown suede coat!” Dagobert accosted Porphyry and asked her to
accompany him to a private room. As soon as Porphyry was in the room, Dagobert closed the door and
stood in front of it. Dagobert did not examine Porphyry’s coat, but started bombarding her with questions,
such as: “How much other stuff have you stolen from us?” “Did you know that this store always
prosecutes shoplifters?” “Are you married?” “How old are you?” Porphyry was frightened and answered
all of Dagobert’s questions. She kept trying to tell him to look at her coat, but every time she finished
answering one question, Dagobert quickly asked another. Finally, after half an hour, Dagobert looked at
the coat and found the Italian label. He knew that all the suede coats sold by Maison Mamselle were made
in Taiwan or Mexico, so he told Porphyry, “Sorry for the mix-up, but you’d be amazed how brazen some
shoplifters are.” Dagobert opened the door and Porphyry left the store.

If Porphyry sues Maison Mamselle for false imprisonment, will she prevail?

(A) Yes, because the detention was longer than necessary to establish that Porphyry had not stolen the
coat. = this one is better because while the shop owner has shopowner privilege, it still has to be

450121682.doc Page 3 of 57
conducted in a reasonable manner. While they satisfied reasonable suspicion, the questions were
irrelevant and the security guard refused to look at the coat immediately to pass the unnecessary
interrogation.

(B) No, because Suzie had reasonable cause to believe that Porphyry had stolen the coat.

(C) No, because the stores are allowed to detain people suspected of shoplifting and to question detainees
about it.

(D) No, because Maison Mamselle should not be held liable for an intentional tort committed by
Dagobert. = no. dagobert didn’t commit it

7. Calvin Calhoun and Sammy Samuelson were members of the Kappa Kappa Kappa fraternity at
University of Houston. During homecoming weekend, the fraternity held a big party which was widely
attended by students and alumni. At the party Calvin and Sammy were “horsing around” when Calvin
pulled out a gun. He put the barrel to Sammy’s head and said “Hey, man, let’s play Russian Roulette.”
Sammy, who had consumed a fifth of Jack Daniels, was very intoxicated. Thinking that the gun was
unloaded, Sammy responded, “Sure, dude.” Calvin then pulled the trigger and the gun made a blank
sound. Calvin and Sammy both laughed and continued partying.

Unknown to Sammy, Calvin had placed two bullets in the gun’s chamber before making his challenge.
The next day, Sammy found out the gun was in fact loaded when Calvin pulled the trigger.

If Sammy asserts a tort action against Calvin, he will most likely:

 Remember that awareness after the fact does not constitute as assault. It needs to be immediate
apprehension.

(A) Prevail, if Calvin intended to place Sammy in apprehension. = even if Calvin intended to place
Sammy in apprehension, Sammy could not have been placed in apprehension without the requisite
awareness.

(B) Prevail, because there was a substantial certainty that Sammy would be placed in apprehension. = no
where in the fact pattern states that Calvin was certain that Sammy would be placed in apprehension..

(C) Not prevail, because he was unaware that the gun was loaded. = the fact pattern states that
Sammy thought it was UNLOADED. Therefore he was not placed in apprehension. There was
no awareness.

(D) Not prevail, because he consented to participate in the Russian Roulette game. = Consent wouldn’t fly
here because clearly,Sammy wasn’t in a position to make coherent decisions so the consent would be
based on misrepresentation?

8. Dino Day, a twenty-year-old college dropout, lived with his parents in a residential section of
Springwood. While his parents were away on vacation, Dino had a party at his home. Throughout the
evening Dino played the stereo at its maximum volume. The music and the load noise disturbed his next-
door neighbors, Mr. and Mrs. Knight. Not being able to sleep, the Knights called the police and
complained about the noise from Dino’s home. About one hour later, the police came to the Day
residence. However, by the time they arrived the party had ended and no noise was emitting from the
house. Nevertheless, the police went to the front door and rang the doorbell. When Dino opened the door,

450121682.doc Page 4 of 57
the officers recognized him as a person who had repeatedly disturbed his neighbors and arrested him.
Dino was taken to the police station and detained overnight.

If Dino asserts a claim for damages for false imprisonment against Mr. and Mrs. Knight, he should:

(A) Recover, because they were responsible for calling the police.

(B) Recover, if the police arrest was improper.

(C) Not recover, because Dino’s conduct constituted a nuisance.

(D) Not recover, if the police arrested Dino on their own initiative.

450121682.doc Page 5 of 57
For Questions 9-10:
Bobbie Kennedy and Jane Smith were recent law school graduates who were studying to take the bar
examination in the state of Ames. On the first day of the bar exam, the young ladies were administered the
Multistate Bar Examination. After both women had completed the first half of the morning section, they went to
the restroom. Bobbie and Jane entered the bathroom almost simultaneously and walked into adjoining stalls. Carol
Hebert, a proctor, who was hired by the Ames Board of Bar Examiners, became suspicious and followed the
women into the bathroom. Suspecting that they were passing notes to each other under the partition, Carol
accused them of cheating when they left their stalls. She then demanded that they both accompany her to the
administrator’s office. When Bobbie refused, Carol grabbed her arm gently but firmly and led her to the office.
Jane followed.

While in the office, Carol opened and looked into each of their pocketbooks. She then demanded that the
two women remove their clothing. Carol carefully went through the clothing, but found no crib sheets or other
evidence of cheating. She then gave both women permission to leave. Although Bobbie and Jane were
embarrassed and deeply humiliated, they returned to their seats and completed the examination.

9. If a claim based on battery is asserted on behalf of Bobbie against the Ames Board of Bar Examiners and
the latter does not claim that it had a privilege to detain her for investigation, Bobbie will probably:

(A) Prevail, because Bobbie did not consent to any touching by Carol. Battery elements: intent to
cause harmful, offensive touching upon another and kwsc that the harmful or offensive
touching will occur. Unpermitted without consent.

(B) Prevail, if Carol failed to exercise reasonable care. = even if she exercised reasonable care, it was still
unconsented touching. Reasonable care is not an element of battery..

(C) Not prevail, because Carol did not use unreasonable force. = the most trivial touching suffices for
battery. As long as it is unconsented or offensive.

(D) Not prevail, if Bobbie did not suffer any physical injury from the touching. = battery does not require
physical injury. Just offensive and unpermitted touching.

10. If a claim based on false imprisonment is asserted on behalf of Jane against the Ames Board of Bar
Examiners, and the latter does not claim that it had a privilege to detain her, Jane will most likely:

(A) Prevail, if Jane reasonably believed she would not be permitted to complete the examination unless
she accompanied Carol. = false imprisonment is intent to contain another person or third party
against their will in a confinement where there is no reasonable means of egress (escape)
Remember that cops have the privilege of arrest but they assume the risk of making a mistake. If they mistakenly
arrest someone, they are liable for any battery, assault or false imprisonment. But Police Officers, like
shopkeepers privilege, do have the privilege to detain someone they have probable cause to believe has engaged
in wrongdoing. BUT they may be charged with FI if they exceed the scope of their authority or if the arrest is
unreasonable (looking for black man, arrests Chinese man) = unreasonable justification or without justification

(B) Prevail, because Jane submitted to Carol’s demand to accompany her to the office. = fact pattern does
not state that jane submitted. If you submit to someones demand then you cannot sue for false
imprisonment. NO CONSENT. CONSENT IS A DEFENSE. Unless you consent and they bring you
out of scope of the consent. Like you say okay I agree to be in the car with you but he ends up not
letting you leave. Or taking you elsewhere.

450121682.doc Page 6 of 57
(C) Not prevail, unless Carol threatened to use physical force against her. = you do not need merely
physical force. You can use threats, hold onto valuable belonging, use walls to keep them contained or
even make them feel as if they are required to be contained against their will.

(D) Not prevail, because Carol did not use any physical force against her. = you don’t need JUST physical
force alone.

450121682.doc Page 7 of 57
11. Keiki, a graduate of University of Hawaii Law School, received notice that she had successfully passed
the Hawaii bar exam. To celebrate passing, Keiki went out with a few friends to Aloha Hut, a popular
campus bar. While sitting at the bar, Malihini, Keiki’s friend, ordered a round of “Hula Girls” for
everyone. A “Hula Girl” is an extremely potent alcoholic drink consisting of 2 oz. of dry gin, 1 oz. French
vermouth with 1 tsp. grenadine. After the drink is mixed, it is then served in a glass with an inverted
lemon peel which is cut at the ends to represent a grass skirt.

Although Keiki had never heard of a “Hula Girl” before, she was in a festive mood and drank it anyway.
A few minutes later, Keiki became very dizzy and attempted to stand up from the bar stool. As she tried to
walk to the restroom, she got nauseous and then vomited over Haole, a customer sitting at a nearby table.
Haole was embarrassed and greatly humiliated by the incident.

If Haole asserts a claim against Keiki, the plaintiff will most likely:

(A) Prevail, because Keiki’s conduct was extreme and outrageous. = no it wasn’t

(B) Prevail, because an offensive touching resulted. = she did not have the intent to offensively touch or
even to touch

(C) Not prevail, because Keiki’s actions were involuntary.

(D) Not prevail, because Keiki was unaware what she was drinking. = you can be unaware of what you
are drinking but aware of your actions thereafter. Here the only claim that Haole can assert is battery.
= offensive touching however, she did not have voluntary actions.

Battery = voluntary act + intent

12. Cruiser and Flash were driving down Main Street in their distinctively painted, carefully-restored old
American automobiles when Cruiser’s vehicle bumped lightly into Flash’s car, scraping away some paint.
Cruiser and Flash both got out of their cars and examined the damage, after which Flash made
disparaging remarks about Cruiser’s car. Cruiser slapped Flash in the face; Flash punched Cruiser in the
nose. Cruiser seized Flash in a half-nelson, and started to ram Flash’s head into the side of Flash’s car. At
this moment, Flash’s brother, Glitter, who had just driven by, leaped from his car, pulled a knife, and held
it to Cruiser’s throat, saying “Let him go or I’ll cut you.” Cruiser released Flash, but subsequently sued
Glitter for assault. What result?

(A) Cruiser wins, because Glitter threatened him with deadly force. = threats with deadly force is allowed
in self defense and defense of others when there is equal/proportional force threatened

(B) Cruiser wins, unless ramming Flash’s head into a car is considered a deadly attack. = Because if
Cruiser had been ramming Flash’s head unreasonably and it appeared as if Flash was ABOUT TO
DIE, then Glitter had the privilege to pull out a knife

(C) Glitter wins, because Cruiser started the fight by slapping Flash. = the initial aggressor isn’t really
relative to the tort of assault. Assault elements: intent to cause harmful or offensive touching
(attempted battery/threatened battery) or intent to place person in immediate fear or apprehension of
that touching. And apprehension occurs. (recall that person must have the awareness in order for there
to be a tort.)

(D) Glitter wins, because he and Flash are relatives. = not relative ha-ha

450121682.doc Page 8 of 57
13. Catherine and Thomas owned adjoining summer cottages on Lake o’ the Pines. One evening Catherine
decided at the last minute to attend a moonlight party on Perch Pier at the far side of Lake o’ the Pines and
asked Thomas if she could borrow his new 15 foot motorboat to cross the lake as it would take her 45
minutes longer to drive around. Thomas agreed. Before going to the party, Catherine stopped briefly at
another pier right next to Perch Pier in order to purchase bread at the neighborhood grocery. While
Catherine was in the grocery store, another boater rammed Thomas’ boat causing about $200 in damage.

In a suit brought by Thomas against Catherine, who will prevail?

(A) Catherine will prevail unless she negligently docked the boat. = similar to the pizza hypo, unless
they were negligent, they are not liable. If she was negligent in docking it maybe it was her
negligent that caused another boat to ram into thomas.

(B) Thomas will prevail and recover the cost of repair to the boat because Catherine exercised dominion
and control over the boat. = she didn’t exercise dominion or control because those elements usually
happen over a long time. She didn’t commit trespass to chattels because buying bread does not exceed
the scope of authority that Thomas gave to her. It is only a minor deviation such as the pizza hypo.

(C) Thomas will prevail and recover the value of the boat before the accident because Catherine exercised
dominion and control over the boat. = why the value? No conversion here.

(D) Thomas will prevail and recover the value of the boat before the accident because Catherine used the
boat for an unauthorized purpose. = why the value? No conversion here.

14. Ted asked Steve if he could borrow Steve’s boat for a day cruise. Steve agreed, but instructed Ted to stay
inside the bay. Ted cruised around the bay, then, because the ocean was so calm outside the bay, crossed
a mile of open ocean to the Blue Oyster Cult oyster bar on the docks at Santa Patagonia Island. While Ted
was docked, another boat passed the dock at an illegally excessive speed and its wake washed into Steve’s
boat, causing water damage to the upholstery of about $150. Ted returned the boat directly to Steve
without further incident.

What, if any, recover can Steve have against Ted resulting from Ted’s use of Steve’s boat?

(A) Steve can recover nothing, because Ted was not responsible for the water damage to the upholstery.

(B) Steve can recover the cost of repairing the upholstery, because it was damaged when Ted took the
boat out of the bay. = Ted was instructed specifically to stay inside the bay. He exceeded his scope of
authority therefore created a risk of any dangers outside of the bay. This fact pattern is more narrow
than the previous where Catherine just stopped to buy bread or the gf just stopped to buy a book. Here
the owner made strict restrictions

(C) Steve can recover the value of the boat, because Ted was in dominion and control of the boat when it
was damaged. = no conversion here

(D) Steve can recover the value of the boat, since Ted violated his instructions about staying in the bay. =
no conversion here

15. One night after Deveraux had been in California for about six months, he missed his girlfriend in Texas so
badly that he wanted to leave California immediately. Lacking cash for the trip, Deveraux approached his

450121682.doc Page 9 of 57
friend Sam for a loan. Sam declined. Deveraux then went to his neighbor, Hugh, and offered to buy
Hugh’s used stove for $50. Deveraux wrote Hugh a check for $50 for the stove. Deveraux then sold the
stove to Sam for $50 and left for Texas. Having discovered that Deveraux’s account was closed and that
Sam had his stove, Hugh sued Sam for conversion. Which of the following facts, if true, would give Hugh
the best chance of recovery against Sam?

(A) Deveraux knew his account was closed at the time he wrote the check to Hugh.

(B) Sam knew Deveraux had closed his bank account.

(C) Sam knew Deveraux had given Hugh a check not covered by sufficient funds. = more narrow than
knowing that Deve just closed his bank account. He would need to know that Deve closed his bank
account AND gave Hugh an insufficient check = fraud. Because then he would be in on the fraud.

(D) Sam intended to make a gift of the stove to his sister.

16. Guy was hiking in the High Sierra in April when a severe late season blizzard set in. He became lost in
the snow and wandered for several house trying to follow Whimpering Creek downstream. Fortunately he
came across a small, seemingly unoccupied cabin which he entered for shelter by breaking the lock and
fell asleep. Several hours later, Basil, the owner of the cabin, roared up on his snowmobile and finding
Guy in the cabin, demanded that Guy leave. When Guy refused, pointing out that the storm was at its
peak, Basil dragged Guy out into the snow, then closed and locked the door. Utterly by chance, Guy
managed to find another cabin, but suffered severe frostbite, leading to the loss of several toes. If Guy
sues Basil, who will win?

(A) Basil, because Guy refused to leave when asked.

(B) Guy, because Basil did not behave as a reasonable person under the circumstances. = it is not about
reasonableness. It’s a privilege of necessity. Private necessity.

(C) Guy, because the Blizzard was a life-threatening situation. = Guy had a privilege of necessity to take
shelter because it was a blizzard outside. In this case, Basil was not allowed to kick him out. It is not a
matter of being reasonable. Guy had privilege to trespass but would be liable for any damages. Basil
does not owe a duty beyond letting him trespass. He doesn’t have to aid him. But he also cannot force
him out or destroy his property.

(D) Basil, because Guy damaged his cabin. = does not state any damage

For Questions 17 – 19:

John and Mary were camping just south of a river in the national forest when a forest fire was ignited
nearby by lightening. In order to avoid the flames, John swam across the river and untied a canoe from a “No
Trespassing” sing, one of many were stuck in the riverbank about every 100 feet. In his haste, John destroyed the
sign in freeing the canoe. John paddled the canoe back across the river to get Mary. John and Mary headed west
hoping to find a public road. Ogre, the owner of the property, was out practicing his bowhunting skills when he
saw the couple and assumed they were intending to camp on his land. An accomplished woodsman, Ogre trailed
them. Intending to frighten them so that they would leave and not return, Ogre shot an arrow so that it would pass
in front of the two campers and “thwack!” into a tree just before they walked by. At the moment of release,
however, another lightening strike occurred very close by, startling Ogre so that his arrow passed closer than he
had intended, narrowly missing Mary and hitting and cutting John’s hand.

450121682.doc Page 10 of 57
17. Can Ogre recover for the damage done to his “No Trespassing” sign?

(A) No, because John was privileged to enter into Ogre’s land to escape the forest fire.

(B) No, because the damage to Ogre’s property was de minimis.

(C) Yes, because John was a trespasser.

(D) Yes, because John destroyed the sign. = even if John was privileged to enter the land, he is still
liable for any damages he accrued while on the land.

18. Can John recover against Ogre for battery?

(A) No, because John was a trespasser. = you are only allowed to use reasonable force to defend your
land. In this case, Ogre used a bow and arrow? Which is not reasonable and it is a deadly force. You
are never allowed to use deadly force to defend your land unless you are faced with equal force.

(B) No, because the lightening was a intervening superseding force. = the lightning was not a superseding
force in that it bars Ogre from battery. Because C

(C) Yes, because Ogre intended to frighten him. = Ogre initially had the intent to place immediate
apprehension or attempt/threaten offensive/harmful contact John. (Assault also needs
awareness and for the apprehension to result) But because he missed and actually hit john, the
intent from assault is transferred over to battery.

(D) Yes, because John was escaping from the forest fire.

19. Can Mary recover against Ogre for assault?

(A) No, because she was not touched by the arrow. = assault does not need actual touching to result

(B) No, because Ogre believed she was a trespasser. = you cant assault a trespasser

(C) Yes, because Ogre used excessive force. = assault does not require excessive force. Just that the actor
intends to bring out harmful or offensive contact (attempted battery or threatened battery) or
immediate apprehension and apprehension results

(D) Yes, since Mary must have seen the arrow pass in front of her. = assault requires awareness and
if mary must have seen the arrow then she must have felt apprehended in the moment

For Questions 20 – 22:

Franklin entered Milton’s Men’s Store at approximately 6:45p.m. to look at some suits that were on sale.
Franklin selected 3 suits from the rack and approached a clerk. The clerk told Franklin they were very busy and he
should wait on himself, so Franklin took the suits to a dressing room.

Signs posted on the walls of the store stated that closing time was 9p.m.; however, because of a special
awards banquet for employees, Milton’s was closing at 7p.m. that evening. The clerks in a hurry to get to the
banquet, did not check the dressing room before leaving.

450121682.doc Page 11 of 57
When Franklin emerged from the dressing room a few minutes after 7p.m. the main showroom was dark
and he was alone. Within seconds, a motion detector noted his presence and set off a silent alarm. Milton, the
store’s owner, heard the alarm, rushed into the store, and grabbing Franklin by the arm, hustled him into the
security offices. Milton ordered Franklin to remain in the office while the police were called.

At this point, Milton remembered that he was expected to be the Master of Ceremonies at the banquet and
having seen no actual merchandise in Franklin’s possession, decided to forget the whole thing. He instructed the
night watchman to just let Franklin find his own way out and left.

When no one returned in more than an hour, Franklin cautiously opened the office door, saw a light at the
end of the store, dashed out, and sped away in his car.

20. If Franklin sues the clerks for false imprisonment for being locked in the store, will the clerks be liable?

(A) Yes, because reasonable clerks would have checked the dressing room before leaving the premises. =
false imprisonment isn’t about whether or not reasonable clerks would check the dressing room

(B) No, because Franklin suffered no damage. = false imprisonment only requires that you were
contained against your will

(C) No, because most patrons leave dressing rooms within a matter of minutes. = is not relevant here
and facts state that he was not in there for an excessive amount of time

(D) No, unless the clerks knew Franklin was still in the dressing room at the time they locked the
store. = If the clerks KNEW that franklin was in the dressing room and left anyway then they
had the intent to keep him confined

21. If Franklin sues Milton for battery, who will prevail?

(A) Milton, because he was acting under a reasonable belief that he was defending his property. = this is a
defense to battery. However, franklin had the privilege to be on site. WATCH OUT.
Well in shopkeepers privilege, te owner has privilege to excervise reasonable force to detain who
reasonably believes is making an attempt to damage or steal. But if its inappropriate or unreasonably
harsh response then it can give rise to assault, battery or fi.
REASONABLE FORCE IS ALLOWED TO DEFEND PROPERTY. However unlike self defense… if
you are mistaken even a REASONABLE MISTAKE against an innocent person, you are liable for the
battery. basically mistake is not excused.

Milton may have reasonably believed that he was defending his property from Franklin and therefore
excercised reasonable force. However, he was mistaken and franklin was innocent so therefore Milton as
a shopowener is liable for the battery.

So like a cop arrest can use reasonable force for a lawful arrest. But if its unlawful, then they can be liable
for battery, assault or FI. They assume the risk of making a mistake.

(B) Milton, because Franklin had exceeded the scope of the consent for him to be on the premises. =
scope of consent is not an element of battery. Also Franklin had consent to be on site.

(C) Franklin, because he did not know the store was closing early that evening. = Franklin not
knowing that the store was closing early means that he had a privilege to be there. It means that
he entered with the consent of the store being opened. Also the clerks left him there. Therefore

450121682.doc Page 12 of 57
he was not trespassing. If he is not trespassing then it doesn’t matter if Milton was under a
reasonable belief that he was defending his property. COMING onto the property / store when
it was open and being locked there after closing hours = you still have privilege to be there.

Milton will be liable for a touching to Franklin whether or not the force that he used was
unreasonable under the circumstances.

(D) Franklin, but only if Milton used force that was unreasonable under the circumstances. = battery can
be trivial touching, does not need unreasonable force . again its not only if he used unreasonable
foce.shopowners can be liable if they used force when they don’t have the privilege. Mistakes are
excused only if it was reasonable.

22. If Franklin sues Milton for confinement in the security office, what will be the likely result?

(A) Milton will win, because he is privileged to detain a person if he has a reasonable belief the person
has committed a theft = he would have won but he left

(B) Franklin will win, because Milton went to the banquet without telling Franklin he was free to leave. =
A would have applied because Milton had a privilege to detain Franklin if he had a reasonable belief
BUT the moment he decided to just leave him there and let him figure it out made it so that he
exceeded his scope of privilege and therefore it became unreasonable containment.

(C) Franklin will win, because there was no reasonable means of escape. = there was reasonable means of
escape…

(D) Milton will win, because he decided not to call the police. = he still left franklin there unreasonably

23. Rellik had a very distinctive, gravely voice, which Mitciv, who was known to his friends as “Weasel,”
knew well. Mictiv won a $500 bet on a football game from Rellik, which infuriated Rellik. That evening,
while drinking in a darkened bar, Mitciv heard a gravely voice in the next booth which he assumed to be
Rellik’s. Suddenly the gravely voice got very loud, shouting drunkenly, “I’m going to pound that little
weasel!” and a figure suddenly loomed out of the next booth. Mitciv, fearing that he was going to be
attacked, jumped up and punched the figure in the stomach, knocking the person down. The bartender
appeared with a flashlight, whereupon Mitciv discovered that he had punched Tom Waitaminit, the
famous, gravely-voiced singer, who happened to be in town for a performance.

Waitaminit sued Mitciv for battery and Mitciv claimed self-defense. What result?

(A) Judgment for Mitciv, if he honestly believed his action was necessary to prevent an attack on his
person.

(B) Judgment for Mitciv, if he reasonably believed such action in self defense was necessary. = BE
CAREFUL. Honestly and reasonably is different. You can honestly believe you are a giraffe.
But can you reasonably believe that.

(C) Judgment for Waitaminit, because an aggressor may never rely on self-defense.

(D) Judgment for Waitaminit, because Mitciv attacked him without provocation. = to Mitciv it was
provocation or enough apprehension…

450121682.doc Page 13 of 57
REMEMBER CURSOUVIER CASE WHERE OLD MAN SHOOTS OFF DUTY COP> HE
reasonably self defended

For Questions 24 – 25:

Jim, an employee of the Hockey League, was assigned by the League to officiate at the championship game in
Selland Arena between the Fresno Flyers and the Portland Speed Demons. The Flyers’ star player, LaFleur,
became irate after Jim assessed a series of penalties against him for unnecessary roughness. Finally, to the home
crowd’s roar of disapproval, Jim ejected LaFleur from the game. Before leaving the ice, LaFleur batted the
hockey puck from Jim’s hands with his stick. The puck sailed into the crowd and broke Charles’ finger. Charles
had entered the arena without a ticket by successfully impersonating a member of the press. The Flyers lost. As
Jim and the other officials walked towards the dressing room, Jim was hit on the back of the head by a bottle
thrown from the crowd.

24. If Jim sues LaFleur, for what tort will he recover?

(A) Assault only because LaFleur did not touch Jim’s person. = its not only because he didn’t touch jim.
Assault can be apprehension.

(B) Nothing, as Jim consented to contact as part of working as a referee. = Not after a time out or game is
called. But be ordinary carelessness. Not reckless or negligent. Also not deliberate, willfully or
wantonly during the game. Consent to accidental incidental game play. Not deliberate.

(C) Battery, because Jim suffered offensive touching to his person. = but he didn’t touch jim directly. He
batted the puck from jims hand so D

(D) Battery, because Jim was holding the puck. = in this fact pattern, you have to connect the
missing piece that Jim was holding the puck and La Fleur batted it out of his hand. Battery can
consititute chattels or close proximity touching so long as it is offensive too. Does not need to
create bodily harm. Does not need to be direct touching so long as its intimate to the person.

25. If Charles sues LaFleur, who will prevail?

(A) Charles, because LaFleur intended to touch the puck. = becase Lafleur intended to bat the puck
out of Jims hands and then it hits Charles head.. the intent (battery) is transferred. Since
theintent was og there.

(B) LaFleur, because Charles is a trespasser. = no

(C) LaFleur, because he did not intend to touch Charles. = does not need to intend to touch Charles. Just
intent to touch and create harmful offensive contact. Can be transferred.

(D) Charles, unless a reasonable person in LaFleur’s position would not expect the puck to fly into the
crowd. = it doesn’t matter if he did not expect the puck to fly into the crowd. When he initiated the
touching (offensive/harmful) of the puck, he bore the risk of it htting into the crowd.

For Questions 26 – 27:

Reynolds owned a sporting goods store specializing in hunting and camping supplied. His Friend
Robinson had often cautioned him that he should not leave so much of his merchandise out in the open. One day
as Robinson entered the store to pay a visit, he decided he’d put his unheeded warnings into action by playing a

450121682.doc Page 14 of 57
joke on his friend. Robinson took a hatchet that was lying on a display case and picked it up over his head as if he
were going to strike Reynolds, who was standing at the register with his back turned. Also, in the store at that time
was Haden, an elderly man who Robinson knew had a weak heart. Robinson noticed Haden standing there.
Robinson then said, “All right buddy, one false move and it’s over.” Haden, thinking Robinson meant what he
said, suffered a heart attack and collapsed.

26. If Reynolds asserts a claim against Robinson the most likely result is that he will:

(A) Recover, if Robinson was negligent.

(B) Recover, if he feared Robinson would hit him. = apprehension

(C) Not recover, if he suffered no harm. = harm doesn’t need to be suffered for assault just need awareness
and imminenet apprehension

(D) Not recover, because Robinson was only joking. = jokes are not a defense

27. If Haden brings suit against Robinson for infliction of emotional distress and seeks for recovery for
damages arising from the heart attack, he should:

(A) Prevail, since Robinson knew Haden was present and had a weak heart. =he was aware of the risks

(B) Prevail, because Robinson assaulted Reynolds.

(C) Not prevail, unless Haden was a member of Reynold’s immediate family.

(D) Not prevail, because the resulting harm was unforeseeable.

For Questions 28 – 29:

Hunter was on his property one day looking for rabbits and other small game which he shot occasionally
for sport. As he rounded a clump of bushes he spotted Caine, whom he thought was a man wanted by the police.
Caine, who had his back facing Hunter, was carrying a rifle on his shoulder. Hunter called out to Caine to stop.
Caine was startled as he turned around and his rifle fell forward so that it pointed directly at Hunter. Huner,
thinking Caine was about to shoot him, fired his rifle at Caine. The bullet missed Caine and hit Able, a trespasser.
Hunter was aware that people often walked onto his land since there was a pond adjoining the property which
provided boating and fishing activities.

28. If Able asserts a claim against Hunter for Battery, Able will:

(A) Recover, because Hunter intended to hit Caine. – he intended to hit caine out of self defense.. but
the intent of self defense can be transferred?

(B) Recover, because Able suffered a harmful and offensive contact.

(C) Not recover, because Hunter accidently shot Able.

(D) Not recover, because Hunter reasonably acted in self-defense. = if one reasonably acts in self defense
and accidentally shoots another, he is still protected by his defense.

29. Hunter asserts a claim for assault against Caine. In his action the most likely result is that Hunter will:

450121682.doc Page 15 of 57
(A) Recover, because Caine pointed the rifle at him.

(B) Recover, because Caine’s act of turning around was voluntary.

(C) Not recover, unless Caine intended to scare Hunter. If the intent to apprehend is not there then no
assault. Assault needs the actor to intentionally cause imminent and immediate apprehension or
attempted batter/threatened battery attempt to make offensive/harmful contact

(D) Not recover, if Caine did not intend to shoot Hunter. Does not need to go all the way as to intend to shoot.
Just intent to imminently and immediately apprehend.

30. Earhardt was a certified airline pilot who owned a single engine Cessna plane. One afternoon she invited
Cass, her girlfriend, to go flying with her. Cass, who was scared of flying, reluctantly agreed. During the
flight, Earhardt decided to play a practical joke and shut off the plane’s engine. When the plane went into
a nose dive, Earhardt said, “Oh my God, we’re going to crash.” Cass became panic-stricken and started
screaming hysterically. A few seconds later, Earhardt re-started the engine and said, “Don’t worry…I was
only joking.” The plane landed safely and neither person was injured.

If Cass brings suit against Earhardt, she will probably:

(A) Recover, if a reasonable person would have been distressed by Earhardt’s actions. – this does not
prove IIED. You need to state that the conduct was extreme and outrageous. Fact pattern already
states that she was distressed. Don’t need to prove if a reasonbale person would be distressed. Need
to prove that the condct was outrageous.

(B) Recover, because Earhardt’s conduct was extreme and outrageous. = fact pattern states shock
and panic. IIED is an intent to place a reasonable person or reckless disregard in emotional
distress by extreme and outrageous conduct. And person does suffer from emotional distress
results.

(C) Not recover, unless Earhardt was aware that Cass was scared of flying. = actor doesn’t need to have
awareness of the partys fears. Actors conduct just needs to be extreme and outrageous to a reasonable
person. The only time you need to think about awareness of special fears is when a third party (family
or non-family that suffers bodily harm) is claiming IIED. Awareness of persons fears falls under the
bystander rule. If the bystander is a family member, actor needs to be aware of their precense. If the
bystander is a stranger, actor either needs to know their weakness and fears and does it anyway =
reckless disregard. Or even not aware, then the person suffers bodily harm.

(D) Not recover, because Cass did not suffer any physical injury. = not required for IIED

For Questions 31 – 32:

Defense Systems Company (hereinafter referred to as Defense) provided home security protection to
property owners in the San Diego and La Jolla area. Chuck Munchkin, who owned a summer cottage in La Jolla,
hired Defense to provide 24-hour protection during the winter months when his home was unoccupied. According
to the security arrangement, Defense’s uniformed guards would periodically patrol the property and, if necessary,
provide an “armed response” to any unauthorized individuals who were found trespassing on the property.

450121682.doc Page 16 of 57
Defense provided security protection to Munchkin’s property for two years. Then on December 7, 1983
Munchkin notified Defense that he was planning to sell the cottage. As a result, he requested that Defense
discontinue its home protection service effective immediately. Two weeks later, Kelly Winslow, a burglar, broke
into the cottage and was ransacking it. As Kelly was running across the front lawn (of Munchkin’s property)
carrying a portable television set, he was seen by Doug Fauts, a Defense security guard. Fauts, who was driving
past the home on security patrol, had not been informed that Munchkin had discontinued protection service. Fauts
suddenly stopped his patrol vehicle and ran toward Winslow shouting, “Stop…Don’t move or I’ll shoot.” Startled,
Winslow dropped the television set and began to flee. Believing that the suspected burglar was about to escape,
Fauts pulled out his service revolver and fired a bullet at Winslow. The shots struck Winslow in the leg, seriously
wounding him.

31. If Winslow asserts a claim against Defense Systems for damages to his injuries, Winslow will:

(A) Prevail, because Fauts used unreasonable force to protect Munchkin’s property.
You are not allowed to use unreasonable threat or force to defend property.

(B) Prevail, because Munchkin had notified Defense to discontinue its security protection service.

(C) Not prevail, because Winslow was trespassing on Munchkin’s property.

(D) Not prevail, because Winslow was burglarizing on Munchkin’s cottage when he was shot.

32. If Winslow asserts a claim against Munchkin for damages for his injuries, Winslow will probably:

(A) Prevail, because Winslow was unarmed when he was shot.

(B) Prevail, if he did not know nor had reason to know that Defense provided an “armed response” to
suspected trespassers. = winslow no longer owes a duty though

(C) Not prevail, because Winslow was committing a criminal act when he was shot.

(D) Not prevail, because Munchkin had discontinued his protection service from Defense when the
shooting occurred. = a landowner has a duty to warn trespassers of any dangers or in this case “armed
forces” but since Munchkin discontinued his service, he was not aware of even having a duty to warn
therefore he is not liable

33. Vic Vicory was an attorney who had an office in Greensboro. After winning a big antitrust case, Vic and a
few associates decided to celebrate and have a few drinks at The Rainbow Room, a popular downtown
watering hole. After having two gimlets (a cocktail containing vodka and Rose’s lime juice), Vic left his
friends and drove home.

Vic, who was a bit tipsy, began driving in an erratic and reckless manner. He was traveling at an excessive
speed along a residential section of town when he approached a sharp curve in the roadway. Trying to
negotiate the turn, Vic lost control of his vehicle and it veered off the road and landed on the front lawn of
Edna Edsell’s property. = unintentional but negligent and reckless

If Edna asserts a claim against Vic for trespass, she will most likely:

(A) Prevail, because Vic was operating his car recklessly.

(B) Prevail, because Vic entered onto her property.

450121682.doc Page 17 of 57
(C) Not prevail, unless Vic damaged her land. If one unintentionally but negligently and
dangerously from an abnormal activity enters land onto another, they are liable if damages
occur.

(D) Not prevail, because he did not intentionally enter onto her property.

34. Carl owned a beautiful tract of land in the Big Bear mountain range. The property was purchased by Carr
and used as a family vacation retreat. About 200 yards of Carr’s property bordered along the Big Bear
Lake shoreline. Ott lived on a stream that flowed along one boundary of Carr’s land and ran into the lake.
When Carr acquired ownership of Big Bear tract, he had a channel dredged across his land from the
stream to the lake at a point some distance from the mouth of the stream. Ott erroneously believed that the
channel was a public waterway. Since the channel served as a convenient shortcut to the lake, Ott made
frequent trips across the channel in his fishing boat. In no way did Ott’s use of the channel cause any
harm or damage to Carr’s property.

After Carr learned about Ott’s use of the channel, he requested that Ott desist further entry onto the
waterway. Ott, who until that time was unaware of Carr’s ownership claim, agreed. Nonetheless, Carr
brought suit against Ott to recover damages for trespass. Judgment for whom?

(A) Ott, because when he used the channel he believed that it was a public waterway.= mistaken belief is
not a defense

(B) Ott, if he discontinued using the channel after learning of Carr’s ownership claim

(C) Carr, but recovery is limited to nominal damages for Ott’s intentional use of the channel. = in
intentional trespass, if you merely have the intent to enter but is a mistake of public land (which isn’t
an excuse) and you don’t accrue any damages, you can be held liable for intentional use of the land
but only to nominal damges. Courts want to allow landowners rights to excercize property rights.

(D) Carr, unless Ott had no other navigable access to the lake. = isn’t really a defense to trespass. Then
everyone can use your land

35. Connie Chung was seven months pregnant and expecting her first child. One afternoon, Connie was in the
kitchen of her home preparing a snack when she suddenly heard the loud screeching noise of automobile
tires. She immediately looked out her window and saw a car driven by Andy Anderson strike Benny Bing,
a nine year-old youngster, who was walking home from school. Anderson was traveling eighty miles per
hour when his vehicle went out of control. The car jumped the curb and hit Benny as he was walking
along the sidewalk in front of Connie’s home. Connie witnessed the incident and saw Bennie hurled 50
feet in the air and land unconscious on her neighbor’s front lawn. As a result of her shock from this
horrible accident, Connie suffered a miscarriage.

If Connie asserts a claim against Anderson, the most relevant issue regarding her right of recover is:

IIED- here you have to make out that its iied.. and look for elements

(A) Whether a person can recover damages caused by shock unaccompanied by bodily impact. There was
bodily impact = miscarriage. Watch out.

(B) Whether a person can recover for damages for harm resulting from shock caused by another’s peril or
injury. This is the issue for IIED…

450121682.doc Page 18 of 57
(C) Whether a person can recover damages based on the defendant’s breach of a duty owed to another
individual.- breach talks about negligence

(D) Whether it is foreseeable that a person may suffer physical harm caused solely by an injury inflicted
on another individual. – prox cause in negligence

36. Scottie entered onto Margo’s property without permission. Margo now sues Scottie for trespass. Which of
the following would provide Scottie with a valid defense?

I. Scottie honestly but mistakenly believed he had Margo’s consent to enter the property. –
mistake is not a defense
II. Scottie’s entry did not cause any damage to the land. – damges are not required for
trespass. So long as intent to enter without permission. = negligent trespass
III. Scottie’s entry onto the property was non-negligent. – if its non negligent it can still
be intentional… or reckless.. = this is a defense to negligent trespass

(A) I only
(B) II only
(C) III only
(D) Neither I, II nor III

For Questions 37 – 38:

Granny Goodridge, aged 72, was riding in an elevator at the Empire State Building in New York City.
When the elevator stopped on the fifth floor, Smokey Robinson entered the elevator smoking a “Panama Red”
cigar. Smokey was standing in front of Granny on the elevator when Granny tapped him on the shoulder. When
Smokey turned around, Granny pointed to the “No Smoking” sign and said, “Excuse me, sir, would you mind
putting that cigar out?” Smokey indignantly responded by inhaling heavily on his cigar, and then blowing a big
puff of smoke into Granny’s face. When the elevator stopped on the next floor, Smokey then departed.

37. If Smokey institutes a civil action against Granny, Smokey will most likely:

(A) Recover for battery.

(B) Recover for negligence.

(C) Not recover, since Smokey’s prohibited conduct would preclude recovery as a matter of law.

(D) Not recover, since Granny’s conduct was customary and reasonably necessary under the
circumstances.

38. In a civil suit brought by Granny against Smokey, the plaintiff will have actions for:

(A) Assault since no actual body contact occurred.

(B) Battery even though no actual physical harm occurred. = smoke constitutes offensive touching.
Harms dignity.

450121682.doc Page 19 of 57
(C) Intentional infliction of emotional distress because of the extreme and outrageous nature of
Smokey’s conduct.

(D) No cause of action.

39. Rider entered a subway car at the 42nd Street station. Since all of the seats were occupied, Rider stood in
the subway car and grabbed a pole to secure his balance. As the subway car was proceeding cross-town,
Rider glanced at a voluptuous blonde girl standing next to him. Suddenly the subway car made an
unexpected stop. Rider momentarily lost his balance and grabbed the blonde girl around the waist (to
avoid falling). Once Rider regained his balance, he removed his hands from the girl’s waist and grasped
the pole again.

In a civil action instituted by the blonde girl against Rider, he will most likely be found:

(A) Liable for battery.

(B) Liable, if Rider mistakenly believed that the girl consented to the contact.

(C) Not liable, since Rider’s conduct was socially acceptable under the circumstances.

(D) Not liable, since the girl was not harmed by the contact. = no harm necessary

For Questions 40 – 41:

Alice is sitting on her front porch watching her husband Bruce, who is mowing the lawn. Carl, who hates Bruce
but is a friend of Alice’s, whose presence is known to him, draws a pistol and threatens to kill Bruce. Alice, who
is pregnant, suffers severe emotional distress as a result of the trauma and soon afterwards has a miscarriage.

40. In an action by Alice against Carl for mental anguish resulting in her miscarriage, Alice will:

(A) Lose, because Carl did not know that Alice was pregnant. = he saw her there and she was a wife so
doesn’t matter

(B) Win, because it is highly probable that Carl’s extreme and outrageous conduct would cause
emotional distress to Alice. Explains the behavior that led up to the result. Remember in the
crazy upside down airplane case… an element that was missing was that noting that the action
was E AND O.

(C) Lose, because Carl’s actions were directed against Bruce, so only Bruce may recover for emotional
distress. = if its to a third party, must be a family member of who D is aware is present or if non-
familial, must suffer bodily injury . here it can be either since she suffered harm

(D) Win, because she is Bruce’s wife. = does not prove that it was causally related

41. Same facts as in question 26, except that Alice is in the living room and does not see the confrontation
between her husband and Carl. After Carl threatens Bruce, Bruce runs into the house and tells Alice what
occurred. As a result, Alice suffers a miscarriage. In an action by Alice against Carl for mental anguish
resulting in miscarriage, Alice will:

450121682.doc Page 20 of 57
(A) Recover, even though she was not present when the threat was made.

(B) Recover, because she would not have suffered the miscarriage had it not been for Carl’s threat on her
husband’s life.

(C) Not recover, since she was not present when the threat was made. IIED YOU NEED TO BE
PRESENT. Cant discover it later.

(D) Not recover, because “transferred intent” is non-applicable in mental distress cases.

42. Clyde Cooch, a prominent judge, lived next door to Lester Biggs. Recently Judge Cooch had sentenced
Lester Biggs’ son, Dopey, to six months in prison on a narcotics charge. One afternoon while judge Cooch
was mowing his lawn, Lester decided to avenge his son’s conviction. Lester set up his water sprinkler
behind some shrubbery separating their adjoining properties. As the judge was mowing his lawn and
came within reach of the water sprinkler, Lester turned on the sprinkling device, and doused the judge
with water.

Judge Cooch would be able to recover against Lester for the which of the following tort(s):

(A) Negligence
(B) Battery
(C) Assault and battery
(D) Battery and trespass – battery for causing the water to spray onto the judge which is offense
and causing third thing to enter the land.

For Questions 43 – 44:

On Thursday morning May 14 two plain clothes Riverdale police officers were shot to death while trying
to arrest a paraplegic bank robber who had failed to surrender to begin a 20-year prison term.

Following the killings, the Riverdale police issued an “all-points-bulletin” for the arrest of Jack Franklin,
a 40-year-old Caucasian, height: 6 feet, weight: 150-155 lbs., and who had been paralyzed below the waist from a
Riverdale bank robbery (that occurred 16 months earlier).

On Friday May 15, Bill Nelson, a security guard at a local department store, was walking down Main
Street in Riverdale when he noticed a tall black man who fit Franklin’s description walking with a slight limp.
Nelson approached the person (and believing him to be Franklin), pulled a gun and arrested him. Nelson held the
man in custody until the police arrived a short time later. The police officer informed Nelson he had arrested the
wrong person. The man had told Nelson that his name was Jones, not Franklin. Jones was humiliated by the false
arrest.

43. If Jones asserts a claim for assault against Nelson, he will:

(A) Succeed, if Jones saw Nelson pointing the gun at him. – assault so remember that you need
awareness for assault

(B) Succeed, if Nelson’s mistaken belief was unreasonable. – even if nelson was unreasonable, there
would be no assault if you didn’t establish awarenss

450121682.doc Page 21 of 57
(C) Not succeed, because Nelson didn’t intend to injure Jones. = assault don’t need intent to injure but
just intent to cause harmful or offensive contact OR imminent apprehension that the contact will
occur

(D) Not succeed, because Jones didn’t suffer any injury from Nelson’s act. = don’t need to suffer injury.
Just imminent apprehension

44. If Jones asserts a claim for false imprisonment against Nelson, Jones will:

(A) Prevail, because he was intentionally detained against his will. = he was intentionally detained
against his will but it was also unreasonable therefore B is better. A would be the answer had the
cop been reasonable. But in this fact pattern it emphasizes on the unreasonableness of it.

(B) Prevail, because Nelson unreasonably believed that Jones committed the crime.

(C) Not prevail, because Jones did not suffer any harm.

(D) Not prevail, since the period of detention was necessary to establish the detainee’s identity.

45. On Tuesday, Ruben purchases a new riding lawn mower from Sears. The next day, Ruben was cutting the
grass in his backyard with his mower when it started to rain. Ruben stopped the mower and turned off the
motor switch. He went inside his house and decided to wait until the rain stopped before mowing the rest
of the lawn. A few minutes later, while Ruben was inside his house, the lawn mower suddenly lurched
forward, rolled down a hill and entered onto his neighbor, Gomez’s property. The mower cut Gomez’s
prize rose bushes, destroying them.

It was later determined that the lawn mower’s motor switch was defective. Even though Ruben turned the
starter switcher to the off position, the mower’s engine did not shut off. Since the mower was built with a
new silent rotary engine, Ruben had no reason to know that it was still running when he went into the
house. Ruben would not have been able to discover the defect by any feasible means of inspection.

If Gomez asserts a claim against Ruben for trespass, will Gomez prevail?

(A) Yes, because the lawn mower entered onto Gomez’s property.

(B) Yes, because Ruben is strictly liable for the damage caused by the defective mower.

(C) No, because Ruben was using the mower for its intended purpose. = does not prove anything. Have to
prove that this was non-negligent and non-intentional trespass. When it is nonintentional and non-
negligent and non-reckless.. it is a products liability case and you are not liable for any trespass or
damages.

(D) No, because the defective motor switch was not discoverable by reasonable inspection. = if it wasn’t
discoverable by a reasonable inspection then ruben was not negligent or liable.

46. Paola and Dixon, who are otherwise unacquainted, happened to be riding the same crowded city bus
during the evening rush hour. Neither Paola nor Dixon was able to find a seat and they, along with about
15 other persons were riding the bus as standees. When the bus braked suddenly, the standing passengers
were thrown together, and Paola, who was wearing very high heeled shoes, began to stumble. Dixon tried
to keep Paola from falling, and in doing so he placed his arm around Paola’s waist.

450121682.doc Page 22 of 57
If Paola sues Dixon for battery, will she recover?

(A) Yes, if Dixon intended to put his arm around Paola’s waist.

(B) Yes, because Dixon touched Paola without her permission.

(C) No, but only if Dixon put his arm around Paola’s waist by accident.

(D) No, because Dixon’s conduct was socially acceptable.

47. Pauline sought psychiatric treatment from Donald, a psychiatrist. During his treatment, which consisted
of hour-long analysis sessions twice a week, Donald, unknown to Pauline, videotaped her. No sound
recording was made of the sessions, but Donald was conducting a study on “body language” and planned
to use the videotapes in those experiments. Pauline learned that Donald had been videotaping their
analysis sessions and brought an action against him for battery.

If Pauline does not prevail as to this theory, it will probably be because:

(A) She did not suffer any injury as the result of Donald’s actions. = no injury needed for battery.

(B) Donald had an implied consent to take the actions he did as part of the patient-physician privilege. =
the implied consent is only re: talking at most if anything. No implied consent to body language or
being recorded.

(C) She did not suffer any offensive touching. = battery needs to be an actor that intends to cause
offensive or harmful touching of another without permission and the touching does occur

(D) Donald intended that his actions would foster medical research. = good deed is not a defense

48. During the Miss Metropolis competition, the judges announced that Wilma was a first runner-up and that
Kerry was the winner. As the auditorium quieted for Kerry’s acceptance remarks, Wilma said loudly,
“You only won because you slept with all of the judges, you slut!” Kerry immediately slapped Wilma
forcefully in the face.

Wilma brings an action for battery against Kerry. Who will prevail?

(A) Kerry, because she was provoked by Wilma’s comment.

(B) Kerry, because a reasonable person would have slapped Wilma under the circumstances.

(C) Wilma, unless Kerry’s slap was totally spontaneous.

(D) Wilma, because Kerry intentionally caused an offensive touching.. all yo need is intent to cause an
offensive touching. Provocation is not a defense. Reasonableness is not a factor. And psontaneiy does
not excuse offensive toucing. So long as there was intent to bring about the action

For Questions 49 – 50:

450121682.doc Page 23 of 57
At home one January night, Scott heard something bang against the bay window in the room where he was
reading the newspaper. When he heard another louder bang, he put down the paper and looked out the window.
He saw eleven-year-old Roman, the neighborhood bully, out back packing a large snowball. Fearful that Roman
would break the window, Scott went out the front door and walked around to the back. When Roman saw him,
Scott said, “I want to talk to you, Roman.” Roman ran in the other direction and jumped over the fence belonging
to Scott’s neighbor Tim. Tim had recently purchased a dangerous Rottweiler as a guard dog. Because it was dark,
Roman landed on a birdbath, knocked it over, and woke up the dog. The Rottweiler attacked Roman, who
received severe lacerations and suffered permanent scarring as a result.

49. Tim brings an action against Scott for trespass. If Scot prevails, it will most likely be because:

(A) Scott confronted Roman in order to defend his property.

(B) Scott did not enter onto Tim’s land.

(C) Roman was the one who made the decision to jump over the fence.

(D) Scott did not intend to frighten Roman onto Tim’s property. Scott did not intend to cause Roman to
enter onto Tims property nor did he have KWSC that roman would jump over and enter tims land. It
doesn’t matter if roman was the one hwo made the decision because you can still make a decision to
enter onto somebody elses property but be forced to enter

50. If Roman asserts a claim against Tim to recover damages for his injuries, Roman will:

(A) Prevail, because Tim may not use a vicious dog to protect only his property.

(B) Prevail, because Tim is strictly liable for injuries caused by the vicious dog.

(C) Not prevail, because Roman was trespassing on Tim’s property.

(D) Not prevail, if Tim had signs up warning about the dog. = a owner who uses a dog or any dangerous
animal to protect needs to have warnings for trespasser

51. Adam drove into the parking lot of Bank and was about to pull into an empty spot when Ben cut in front
of him with his automobile and took his parking place. Adam and Ben each got out of their car and Adam
started to yell at Ben. After a heated argument, a fight broke out between them. Calib came out of the
bank at that moment and saw that Adam was getting the better of Ben in the fight. Calib ran to his car,
took a gun from the glove compartment, pointed it at Adam and said, “Stop this minute or I’ll shoot.”

If Adam asserts a claim against Calib based on assault, who will prevail?

(A) Adam, because Calib threatened him with deadly force.

(B) Adam, because Calib was unaware of who was the aggressor.

(C) Calib, because Adam was the original aggressor by starting the fight with Ben.

(D) Calib, if it was apparent that Adam was about to inflict serious bodily harm to Ben. – defense of
others is a privilege when it is apparent that Adam was about to seriously injure Ben. So in this
instance deadly force is allowed

450121682.doc Page 24 of 57
52. Danny is a 10 year-old, and although his parents make sure he is home by 10p.m. during the school week,
they do not really enforce any sort of curfew on their son during the weekend. One Saturday night, when
Danny ‘s parents went to bed at midnight, Danny was still out. About 2a.m., Danny and two other
children were arrested by the police for breaking the windows and causing other damage to Bernadette’s
antique Cord automobile.

In a suit by Bernadette against Danny for the damages he caused to her automobile, Bernadette should:

(A) Prevail, because Danny, at the age of 10, should have been aware of the consequences of his
action. = remember that in intentional torts, children are tried as adults so long as they can form
intent

(B) Prevail, because Danny deliberately damaged her car. = children are measured to adults. In
negligent torts, children are measured by age, intelligence, education and experience

(C) Not prevail, because Danny is presumed to be under the care of his parents, and therefore, is not
legally responsible for his tortious conduct. = parents are generally not vicariously liable for their
childrens torts unless they were somehow directly responsible or if they were directly under their
care

(D) Not prevail, unless she can show that Danny was mature enough to be aware of the consequences
of his action. = In intentional torts, Danny just needs to form intent. Not maturity.

53. Paine and Duncan were playing tennis. Duncan became highly irritated because every time Duncan
prepared to serve, Paine started talking loudly. Paine’s loud talk distracted Duncan from his game, and
Duncan usually faulted on his serves. Duncan told Paine to “cut it out,” but Paine persisted in the
behavior.

Standing several feet away, Duncan swung his tennis racket toward Paine’s head. However, Duncan
slipped as he swung the racket and it flew out of his hand as he lost balance. The racket flew through the
air and struck Paine in the head.

Has Paine grounds for a battery action against Duncan?

(A) Yes, if Duncan intended to create a reasonable apprehension in Paine. = If he intended a


reasonable apprehension.. the intent can be transferred from a possible assault case to
battery

(B) Yes, because the racket struck Paine.

(C) No, because Duncan did not intend the racket to Strike Paine. = you don’t need intention for the
racket to hit Paine
(D) No, but only if Duncan can prove that the owner of the tennis court had not maintained the court
properly and this caused Duncan to slip.

54. Digby was employed by the collection department of Cardshark, Inc., a wholesale distributor of greeting
cards. He was assigned to collect money owed on a consignment account by Mary, the elderly sole
proprietor of a card shop named “Sensitive Greeting Cards.” Digby strode into the shop while Mary was
waiting on some customers and demanded to know why she had not paid her account. Mary, distressed by
his intrusion, assured him that her account was fully paid and asked him to wait until her customers had
left to continue the discussion. Digby, undeterred, continued in a loud voice, “We’re not going to put up

450121682.doc Page 25 of 57
with freeloaders like you anymore. I’m going to come back with a truck and repossess your entire
inventory if you don’t pay up.” Mary, in tears, begged Digby to leave but he only became angrier.
Pounding his fist on the counter, he shouted, “And then we’re going to prosecute you for fraud and put
you in jail!” Digby then left, but by then Mary was severely distraught.

If Mary sues Digby for her emotional distress, who will prevail?

(A) Mary, if Digby’s conduct was extreme and outrageous.

(B) Mary, if she felt subjected to a threat of physical injury. = IIED does not require that you are
subjected to a threat of physical injury. Just that the actor intended or knew with substantial
certainty that sed may result. through his conduct that is Extreme and Outrageous and that he was
aware that it would. And SED results. or was recklesss

(C) Digby, if he did not intend for Mary to suffer severe distress. =REMEMBER that iied does not
need physical injury but that the actor intended or knew with substantial certainty that P would
suffer SED and that his conduct be exreme and outrageous and IF HE DID NOT INTEND WITH
SC THEN he was recklessly disregarding by doing his act. A third party can sue for iied by being
present (indirect Platintiff) if they are 1) a family member and D is aware of their presence 2) a
non family member or stranger but bodily harm need to result. OR D is aware of their presence
and their susceptibilities and harm results

(D) Digby, if Mary owed the money that Digby demanded.

55. Cisneros was sailing down the deep water channel to Lake Washington using the kicker on his sailboat
when LeBeau roared by in his cabin cruiser at about 20 knots, nearly swamping Cisneros. Cisneros raised
his middle finger in the timeworn salute of the impotently angry and shouted a few well-chosen
references to LeBeau’s anatomy and ancestry. LeBeau happened to glance back at Cisneros during the
latter’s tirade and, perceiving that Cisneros was being less than complimentary, swung about and pulled
alongside the much smaller sailcraft. The two skippers exchanged further hostilities centering on each
other’s lack of seamanship, whereupon LeBeau powered away, made a tight circle, and steamed at high
speed directly at Cisneros’s bow. Cisneros was convinced that the boats would collide, so he steered
close to the edge of the channel and abruptly ran aground on a shallow sand bar. LeBeau’s mocking
guffaws rose above the rumble of his engine as he sailed away from the stranded but uninjured Cisneros.

If, after freeing his craft, Cisneros brings an appropriate action against LeBeau for damages, the probable
outcome will be:

(A) Cisneros will win, unless he was himself responsible for provoking LeBeau during the relevant
events.

(B) Cisneros will win, because he believed that LeBeau’s maneuvers threatened imminent
danger of harm to him. – LB can be liable for assault for his voluntary act that intended to
cause or bring about harmful/offensive contact or imminent apprehension of such contact
and where imminent apprehension of such contact results. assault requires awareness by P
which is present in this case.

(C) LeBeau will win, because Cisneros suffered no physical injury of property damage.

(D) LeBeau will win, unless Cisneros suffered severe emotional distress from LeBeau’s conduct. =
notice that Cisneros has multiple torts that he can claim. Notice that when you first read this

450121682.doc Page 26 of 57
question you thought of IIED but answer D restricts Cisneros to only having a claim under iied.
Remember that he can also get LB on assault here. And he doesn’t need to prove SED.

For Questions 56- 57:

As soon as Wanda moved into her new house, she went to the local animal shelter and selected two dogs
to keep her company. The dogs were housebroken but barked constantly at birds and squirrels in the yard. Their
barking was particularly incessant during the day, while Wanda was at work. Morlock, who lived next door and
worked nights, was aggravated by the constant barking, which disturbed his sleep, and decided to let Wanda know
how he felt. One evening, upon learning that Wanda was entertaining her boss and several clients, Morlock came
to her front door with a tape recorder and an electrically amplified bullhorn. He started playing a tape of the dogs
barking, putting it at full volume and amplifying it with the bullhorn. When Wanda came to the door, he began
yelling at her through the bullhorn and berating her in front of her guests for having no consideration for her
neighbors. Wanda, very upset, slammed the door shut. The door struck the bullhorn and jammed it against
Morlock’s face, knowing out two of his teeth.

56. If Wanda asserts a claim based on intentional infliction of emotional distress against Morlock, what will
be the probable result?

(A) Wanda will prevail, because Morlcok’s conduct was extreme and outrageous.

(B) Wanda will prevail if she suffered pecuniary harm from Morlock’s conduct. = you don’t need actual
harm to prove iied

(C) Morlock will prevail, because Wanda suffered no physical harm. = again you don’t need to suffer
physical harm

(D) Morlock will prevail if the barking from Wanda’s dogs constituted a nuisance. = nuisance is a thing or
activity that persists over a period of time. It is usually of intangible material such as vibrations,
sound, anything that relates to the sensory. The person involved in the nuisance does not need to have
the intent to create the nuisance for others. It may even be out of good intentions (such as the
neighborhood that welcomed homeless people) But if it persists over a period of time and begins to
disturb the use and enjoyment of land or interest in land for owners, then courts may weigh the social
utility of the nuisance and the harm done to judge whether or not an injunction or damages will be
granted.Typically smoke and gas does not amount to trespass unless it causes substantial damage.

57. If Morlock asserts a claim based on battery against Wanda, will Morlock prevail?

(A) Yes, unless Wanda did not foresee that the bullhorn would knock out Morlock’s teeth.

(B) Yes, if Wanda knew that the door was substantially certain to strike the bullhorn. = better than A
because she didn’t need to see that the bullhorn would knock out Morlocks teeth.. just that an
offensive touching would result and she needed to be certain that it would.

(C) No, because Wanda was entitled to use force to protect herself.

(D) No, if Morlock’s conduct provoked Wanda’s response.

58. Kim and Kendall were playing handball as part of a Special Olympics-type activity at a residence for
moderately emotionally disturbed adults. Although Kendall suffered from a mental illness, the counselors
had discovered that he was very proficient at handball and had emphasized this activity to raise his self-

450121682.doc Page 27 of 57
esteem. On this particular day, Kendall was winning handily but had become quite agitated by the
competitive aspect of the event. Suddenly, Kendall turned menacingly towards Kim and returned the ball
with great force directly at her. The ball hit Kim in the head, causing her a concussion. If Kim sues
Kendall for battery, will she prevail?

(A) No, because a mentally ill adult is deemed by the law incapable of forming the intent to injure
another.

(B) No, because a mentally ill adult is deemed incapable of forming the intent to touch another.

(C) Yes, because a mentally ill adult is expected to behave as a reasonable person under the
circumstances.

(D) Yes, unless the mental illness prevents Kendall from having the ability to form the intent to touch. = it
is not that mentally ill adults are expected to behave as a reasonable person under the circumstances.
But that they are capable of forming intent. So unless kendall had a disability that kept her from being
able to form the intent to touch then she can be liable since she had the intent

For Questions 59 – 60:

Dortmunder, age 19, was scrawling gang slogans on the fence surrounding Peltier’s yard when Peltier
appeared and said, “Hey! Stop that – you’re ruining my fence!” Dortmunder finished what he was writing and
began to walk away. When Peltier muttered to himself, “Now I’m going to have to repaint the whole fence,”
Dortmunder stopped, turned and said, “Repaint my work and you’re going to have a fire at your house – while
you’re home.”

59. Peltier brought an action against Dortmunder for assault. What result is likely?

(A) Judgment for Dortmunder, because he did not threaten any immediate physical harm. = here in
the fact pattern, the boy says “repaint my work and youre going to have a fire at your house
while youre at home” signifies an unspecified time. Which is not immediate therefore Peltier
cannot apprehend any immediate harm. C is wrong because even if the boy intended that
Peltier experience apprehension of a harmful contact, the intent is there but the immediacy is
not

(B) Judgment for Dortmunder, because any threat he made was conditional and thus avoidable.

(C) Judgment for Peltier, if Dortmunder intended that Peltier would experience apprehension of a
harmful physical contact. = there needs to be an immediate

(D) Judgment for Peltier, if Dortmunder’s threat is considered extreme and outrageous conduct. = not an
element of assault

60. Peltier brought an action against Dortmunder for infliction of emotional distress. What result is likely?

(A) Judgment for Dortmunder, because he did not threaten any immediate physical harm.

(B) Judgment for Dortmunder, so long as Peltier experienced no physical harm as a result of
Dortmunder’s actions. = physical harm not required for iied

450121682.doc Page 28 of 57
(C) Judgment for Peltier, if Dortmunder intended that Peltier would experience apprehension of a harmful
physical contact. = this is assault

(D) Judgment for Peltier, if he suffered severe emotional distress as a result of Dortmunder’s actions. =
notice that iied questions involve a lot of the P actually suffering from SED.

61. Phillip is the owner of a small winery. For several years he has been dumping grape sludge that is the
waste product of the fermenting process in a pit about 100 yards from Jennifer’s property. The sludge
continues to ferment giving off a highly pungent and offensive odor. Jennifer finds it impossible to enjoy
any outdoor activities in her yard because of the odor. If Jennifer’s four-year-old daughter ran onto
Phillip’s land chasing a butterfly, fell into the sludge pit, and was injured, would she recover?

(A) Yes, but only if Phillip knew of her presence. = it doesn’t matter if he knew of her presence. He can
know of her presence but not know that they are likely to fall or be injured.

(B) Yes, if Phillip knew young children had entered his land before and were likely to be injured. = if he
knew that young children entered his land and were likely to be injured then he has a duty as a land
owner to prevent those likely injuries from happening especially to young children who are unable to
form the capability necessary or awareness to know not to enter.

(C) No, because she came on to the land to look at the butterfly, not the sludge pit.

(D) No, because the sludge pit is not a nuisance per se.

62. Barbara worked as an associate lawyer in Triptolemus’ firm for several years, working extremely hard and
winning many important and lucrative cases. Triptolemus then discharged her and took Skip, a recent
graduate of a prestigious law school whose father was a client of the firm, as a partner. Barbara resolved
to get even somehow. She learned that a wall of Triptolemus’ private office bathroom faced the sidewalk
of a busy city street, and had a full length mirror on the inside. One weekend night, Barbara and two
friends used her key, which she had not yet surrendered, to enter the law offices and then picked the lock
of Triptolemus’s private office. Barbara then replaced the bathroom mirror with a two-way mirror and
installed a window in the wall behind it so that a person standing on the sidewalk could see all parts of the
bathroom. Finally, Barbara covered the new window with a curtain. She did not actually want to expose
Triptolemus’ bathroom behavior to the public, but intended that he would believe sometone might have
done so by pulling aside the curtain at a specific moment.

The following Wednesday, Triptolemus’ son visited him at his office and during the course of the visit
used the private bathroom. Alerted by the unusually loud street noise, the son investigated the mirror,
pried it loose from the wall and discovered the curtained window to the sidewalk. When his son called
him into the bathroom and showed him the two-way mirror and window, Triptolemus, who as a cross-
dresser liked to wear a bra and woman’s panties beneath his expensive Italian three piece suits, was so
affected by the thought of the public display of his toilette that he suffered a heart attack.

In an action by Triptolemus against Barbara for infliction of mental distress, the fact that Triptolemus’ son
discovered the mirror-window arrangement and showed it to Triptolemus has what legal effect?

(A) It will not affect Barbara’s liability, because Barbara intended that Triptolemus would suffer mental
distress by discovering the mirror and window. = it doesn’t bar the fact that Barbara intended for
Trptolemus to suffer mental distress. She intended to cause or knew with substantial certainty that it
would cause. If intent upon the D isn’t there then look at recklessly disregarding.

450121682.doc Page 29 of 57
(B) It will not affect Barbara’s liability, because the contributory negligence of the son will not be
imputed to Triptolemus. = . Assuming that the son's actions were negligent, such negligence
would be analyzed under causation principles of foreseeability, not as contributory
negligence. Contributory negligence has no applicability here, since Triptolemus is the
plaintiff and did nothing but suffer injury.

(C) It will supersede Barbara’s liability, if the son was negligent in showing the mirror and window to his
father in that abrupt manner.

(D) It will supersede Barbara’s liability, because the son’s actions were the cause in fact of Triptolemus’
injury. = it wasn’t. the cause in fact was barbara’s set up

63. Shy, a 16-year-old, took his date for the high school prom, Date, out to dinner before the prom. It was the
first time Shy had ever been to a restaurant without his parents, and the first time he had ever been to such
an expensive restaurant. Shy had badly underestimated the amount that dinner would come to, especially
since Date ordered one of the costliest entrees, so he only had the change in his pocket left to tip the
waiter after paying for their meal. Their waiter, Waiter, was a particularly belligerent and supercilious
person who, when he saw that Shy had left a few coins for his tip, grabbed the coins from the table and
threw them in Shy’s face, shouting, “You miserable, pimple-faced bumpkin! How dare you insult me by
leaving this pittance as my gratuity?” When Shy attempted to whisper an explanation, Waiter continued
loudly, as everyone in the crowded restaurant looked on “I don’t want to hear your protestations of
poverty, you oaf. I could tell from your grotesque manners and from the way you’re dressed that you are
an uncouth, unlettered troglodyte.” When Shy attempted to leave, Waiter pushed him roughly to the floor,
exclaiming “Pick up your ridiculous money!” Shy grabbed at the coins on the floor and left, humiliated.

Unknown to Shy, his mother, Mother, had booked a table in a secluded portion of the same restaurant so
that she could videotape his first date. She observed the entire episode with Waiter, suffering severe
emotional distress and as a consequence, subsequently brought suit against Waiter for intentional
infliction of emotional distress. What result is likely?

(A) Judgment for Waiter, because he was unaware of Mother’s presence in the restaurant. = if he did not
know that the mother was present then he did was not able to intend to cause SED upon the mother.

(B) Judgment for Waiter, because Mother was not close enough to have been threatened with any physical
harm from Waiter’s conduct. = you don’t need to be threatened with any physical harm for IIED. Just
that the actor intended to cause or knew with substantial certainty that his conduct would cause SED.
If no intent to cause then must have reckless disregard. That the conduct was extreme and outrageous
and that the P suffered SED as a result. If a third party would like to sue, then it must be a family
member (not after the incident) whose presence is known to the waiter or if it’s a non family member
then he has to suffers bodily injury.

(C) Judgment for Mother, because she suffered severe emotional distress as a result of watching Waiter
humiliate Shy. = even if she suffered SED, the waiter needs to have known that she was there..

(D) Judgment for Mother, if Waiter’s conduct was extreme and outrageous. = the waiter WAS extreme
and outrageous but he wasn’t aware of themother being there so idk if mother can claim

64. Boats-R-Us, Inc. (BRU), a corporation that manufactured and sold luxury yachts, held its annual
shareholders meeting at a dockside amphitheater at its headquarters on the Hawaiian island of Molokai.
Portsmouth, the CEO of BRU, always opened such meetings by debarking directly from one of BRU’s

450121682.doc Page 30 of 57
latest and most expensive models. Just as Portsmouth was about to step onto the gangplank leading from
the yacht to the dock, Dernier kicked the gangplank so that it slid sideways about a foot. Portsmouth, his
eyes on the assembled shareholders in the amphitheater, stepped into the empty air formerly occupied by
the gangplank and plunged into the water. An expert swimmer, Portsmouth quickly clambered onto the
dock, unharmed physically but red-faced with humiliation at having been made to appear a clumsy
buffoon before his fellow executives and the important shareholders assembled for the meeting.

Portsmouth subsequently brought an action against Dernier for damages arising from the humiliation
caused by the dunking. Which of the following is the most probable result? IIED?

(A) Judgment for Dernier, because Portsmouth experienced no physical injury as a result of Dernier’s
actions. = physical injury not necessary for iIED

(B) Judgment for Dernier, if Dernier believed in good faith that moving the gangplank was a harmless
practical joke. = practical jokes are not an excuse

(C) Judgment for Portsmouth, if Dernier was substantially certain that moving the gangplank would cause
Portsmouth to step into the water. = if he was certain then he had the intent for Portsmouth to fall into
the water causing him either distress or harmful offensive contact.

Assault: a voluntary act with intent to cause or bring about harmful or offensive contact or immediate
apprehension of such contact and apprehension of such contact does result.
Battery: a voluntary act with intent to cause or bring about harmful or offensive contact to P’s person or
P’s dignity or any intimate object close to P or cause a thing or third person to bring about harmful or
offensive contact (throwing a rock at somebody/ grabbing something close to body/slapping plate out of
hand/dignity) and if he was certain that such contact would result and such contact does result.

(D) Judgment for Portsmouth, if Dernier was negligent in moving the gangplank as Portsmouth was about
to step onto it.

For Questions 65 – 66:

Gerd was prospecting for gold on public lands in the mountains of the high desert one summer when he
noticed a mountain lion on a nearby ridge. The mountain lion simultaneously noticed Gerd and his pack donkey.
Examining the lion through binoculars, Gerd saw that it appeared to be wounded and was definitely limping.
Knowing that a wounded predator would put aside its normal aversion to a human, Gerd hurried away down the
canyon, only discover that his way was barred by a chain link fence. There was an unlocked gate, so Gerd hurried
through with his donkey. At that moment, Naem, the owner of the land, pulled up in a pickup, and ordered Gerd
off of his property.

65. Gerd explained about the wounded lion, but Naem was adamant. When Gerd refused to go back into the
canyon, Naem summoned police and had Gerd arrested for trespassing. Naem subsequently brought an
action for civil trespass against Gerd. Which of the following is most likely to have a determinative effect
on the outcome of this litigation?

(A) The mountain lion, which was not in fact wounded, ran in the opposite direction as soon as Gerd was
out of sight.

(B) The existence of the fence would place a reasonable person on notice that the land beyond was
private.

450121682.doc Page 31 of 57
(C) A reasonable person would have believed that the mountain lion presented a threat to his safety. If a
reasonable person would have believed that the mountain lion posed a threat to their safety then they
have the privilege to trespass onto somebodys land.

(D) The gate in the fence bore a large sign warning that beyond was private property and trespassing was
forbidden.

66. Assume for this Question only that when Gerd refused to leave his property, Naem opened the gate and
shooed the donkey out, then padlocked the gate shut. The mountain lion, wounded and driven by
desperate hunger, killed the donkey. Can Gerd recover the value of the donkey in an appropriate action
against Naem?

(A) No, because whatever Gerd’s rights to personal safety, Naem had no duty to permit the donkey onto
his property.
(B) No, because Gerd was a trespasser, and had no right to traverse Naem’s land.

(C) Yes, since Naem intentionally or recklessly caused the destruction of the donkey. = when you have
the privilege of necessity, private necessity, then you have the privilege to trespass due to dangers
outside. You are allowed onto the land with your property. And the owner cannot eject you or your
property. He does not have to aid you further.

- Recall that if your property or the thing youre preserving is less than the property you are
damaging?? Or trespassing onto.. then you do not have this privilege.
- Recall that if you are stuck somewhere with others, your luggage /property is then able to be
ejected in order to make room for human lives
- If you pose a threat to multiple people then they can eject you..

(D) Yes, because Naem had an affirmative duty to protect Gerd and his property from the mountain lion.

67. Josephine owned a beautiful black Siamese cat named Ebony. The cat had a value of $50. Josephine
allowed the cat to roam loose in the neighborhood. Ebony frequently entered the backyard of Berk, who
lived next-door to Josephine. Berk hated felines. One day Berk looked out his kitchen window and saw
Ebony defecating on his prize rose bushes.

Berk Telephoned Josephine and told her to retrieve her cat or he would kill it. Josephine immediately ran
over to Berk’s property to get Ebony. In the interim Berk loaded his shotgun and went outside. Josephine
saw Berk with the shotgun and said, “Please don’t shoot my cat.” Berk responded, “I’m sick and tired
cleaning up her crap, maybe you should have toilet trained her better.” Berk then pointed the rifle at
Ebony and shot her. Seeing her cat being shot, Josephine was overcome with grief and became
emotionally distraught. Fortunately, Ebony survived the shooting but suffered a gunshot wound to her leg.

Josephine brings suit to recover damages against Berk. Which of the following causes of action would
afford Josephine her maximum recovery?

(A) Battery. = not to a cat

(B) Conversion = would only give her 50

(C) Trespass to chattels.

(D) Intentional infliction of emotional distress.

450121682.doc Page 32 of 57
For Questions 68 – 70:
While relaxing at poolside one Sunday afternoon, Dickie was struck by a golf ball driven by Marty, a 14
year old boy, who was playing the 8th hole at the Pike Creek Golf Club. The fairway for the 9th hole was 65 feet
wide and 437 yards long, with a dogleg in an easterly direction. Between the fairway and Dickie’s property was a
“rough” containing brush and low lying trees.

As Marty was approaching the green, he hit a towering shot which deflected off a tree, struck Dickie,
bounced off his head and knocked a straw hat off of his girlfriend Patty’s head. Although the ball did not strike
Patty herself, she became startled and fell from her beach chair, thus breaking her arm.

At trial plaintiff offered uncontested evidence that golf balls from the Club’s links regularly traversed
onto his property two to three times a day.

68. Which of the following statements is most accurate regarding the liability of the Pike Creek Golf
Club/Marty for trespass?

(A) Defendants are not liable, since they did not intentionally cause the golf ball(s) to traverse onto the
plaintiff’s property.

(B) Defendants would remain liable for the unpermitted intrusion of the golf ball(s) onto the plaintiff’s
property. = if the fact oattern said the golf course was aware

(C) Since the plaintiff should have reasonably anticipated that living next to a golf course would result in
stray golf balls landing on his property, defendants would not be held liable.

(D) Since the golf balls did not substantially interfere with the plaintiff’s use and enjoyment of his land,
defendants would not be held liable.

69. Which of the following would be Dickie’s proper cause of action against Mary as a result of the golf ball
hitting his head?

(A) Assault but not battery.

(B) Battery but not assault.

(C) Assault and battery.

(D) Neither assault nor battery, since Marty did not intentionally cause Dickie to be struck. = Neither
because here marty did not intend for the balls to fly over andcause damage

70. If Patty initiates a suit against Marty to recover damages for her broken arms, Patty will:

(A) Recover for assault only.

(B) Recover for battery only.

(C) Recover for assault and battery.

(D) Not recover. = neither assault or battery because marty did not have the intent to cause harmful or
offensive touching. Intent to cause harm can be reduced to intent for the act itself when it is in a

450121682.doc Page 33 of 57
socially accepted setting – romeo cannot hug a girl randomly because socially it is not acceptable but
offensive. So here he does not need the intent to cause a harmful or offensive touching. He just needs
the intent to do the act itself. The awareness to do. But in a public bus setting a creep who stares at a
hot girl when the bus shakes at a halt and he loses control, and grabs onto the girls waste, this is not a
battery because it is socialy acceptable. Battery can be where touching occurs thru smoke or some
way where it hurts the persons digity as well. Either way the D needs to have intent and an awareness
or certain that the ball would fly over

Some Explanations for Packet 1

Please note: Not all questions have explanations. An Answer Key follows these
Explanations.

1. (C) is the best answer. The intent necessary for the intentional tort of battery upon Belinda can
be transferred from the intent to commit the same tort upon Seapnor or from the intent to commit
assault upon Seapnor. Finn can be liable even if he did not desire or have knowledge to a substantial
certainty that a touching would occur to Belinda. (A) is incorrect. Finn can be liable for battery even
where he does not know to a substantial certainty that he will touch Belinda. The intent to touch
Belinda is transferred from his intent to place Seapnor in apprehension of an imminent harmful

450121682.doc Page 34 of 57
touching. (B) is incorrect. Where an individual comes to the defense of another, even in circumstances
where a reasonable person would conclude that the other has a right to defend himself, the intervenor
bears the risk that he is mistaken. Thus, the reasonableness of Finn's actions does not protect him from
liability for battery where Martin was in fact the aggressor. (D) is incorrect. Choice (D) reflects Finn's
potential liability on a negligence theory, not in battery.

2. (D) is the best answer. As the professor in charge of the classroom, Seapnor owed a duty to her
students to conduct the class in a manner that would not pose unreasonable risks to them. Since the
facts tell us that numerous students in the class legally carried guns and that Seapnor knew this fact,
Seapnor's use of a teaching device which would give the illusion to reasonable people that someone
was in danger of imminent injury would pose a serious risk that some well-meaning student would try
to assist and someone could be injured. Seapnor breached her duty of reasonable care under the
circumstances. (A) is not correct. Seapnor could still be liable on a negligence theory if she failed to use
reasonable care even in the emergency of attempting to evade the shot. (B) is not correct. Seapnor
could be negligent in staging the scene as well as in how she attempted to evade the shot. Thus, it is not
accurate to say that Belinda could prevail only if Seapnor was negligent in evading the shot.(C) is not
the best answer. While Finn is an intervening tortious force between Seapnor’s original negligent act of
staging the scene and Belinda's injury, such a force is not superseding of Seapnor's liability if her
actions increased the risk of Finn's intervention. By staging the scene, Seapnor created the risk that
someone would misinterpret it and try to intervene.

3. (B) is the best answer. To recover for intentional infliction of emotional distress, Johnson must
demonstrate that Martin committed extreme and outrageous conduct intended to cause or done in
reckless disregard of causing extreme emotional distress which in fact does cause such distress. To
answer a two-tier question such as this one, the student must determine whether each statement in the
first tier is true or false. In this instance, the student is looking for statements of required elements of
recovery for intentional infliction of emotional distress. I is false because there are circumstances in
which a bystander can recover for intentional infliction of emotional distress without showing that
Martin specifically intended to cause Johnson extreme emotional distress. II is a necessary element. III
is false because physical harm is not required for recovery in intentional infliction of emotional distress.
It is a requirement for negligent infliction of emotional distress in a majority of jurisdictions. IV is a
necessary element. Therefore, choice (B) is the best answer because it includes the two stated
necessary elements, II and IV.(A) is not the best answer. Ill is included and physical harm is not
necessary to the tort.(C) is not the best answer. I is included and Martin need not specifically intend to
cause Johnson severe emotional distress.(D) is not the best answer. It includes III and physical harm is
not necessary to the tort.

4. (B) Gilda has committed a battery because she intentionally caused an offensive contact to
Parker. In order to establish a prima facie case for battery, the following elements must be proved: (i) an
act by the defendant that brings about harmful or offensive contact to the plaintiff’s person: (ii) intent
on the part of the defendant to bring about harmful or offensive contact to the plaintiff's person; and
(iii) causation. Certainly a reasonable person of ordinary sensibilities would consider contact with the
gas to he offensive. The defendant is liable not only for "direct" contact, hut also "indirect" contact; i.e.,
it will be sufficient if she sets in motion a force that brings about harmful or offensive contact to the
plaintiff’s person. Thus, even though Gilda did not have direct contact with Parker, she allowed the
chemical fumes to pass across the sidewalk and thereby caused the offensive contact to Parker. The fact
that she wanted to replicate the class experiment and therefore went to an area with heavy pedestrian

450121682.doc Page 35 of 57
traffic evidences her intent to bring about the offensive contact. (A) is incorrect. In order to establish a
prima facie case for assault, the following elements must be proved: (i) an act by the defendant creating
a reasonable apprehension in plaintiff of immediate harmful or offensive contact to plaintiff’s person;
(ii) intent on the part of the defendant to bring about in the plaintiff apprehension of immediate harmful
or offensive contact with the plaintiff’s person; and (iii) causation. Here, Gilda’s actions created actual
contact but did not create an apprehension of contact on Parker’s part; he was evidently not aware of
the gas until he came into contact with it. Hence, the applicable tort is battery, not assault. (C) is
incorrect. To establish a prima facie case for intentional infliction of emotional distress, the following
elements must be proved: (i) an act by defendant amounting to extreme and outrageous conduct; (ii)
intent on the part of defendant to cause plaintiff to suffer severe emotional distress, or recklessness as to
the effect of defendant’s conduct: (iii) causation; and (iv) damages. Actual damages are required. But it
is not necessary to prove physical injuries to recover. It is. however, necessary to establish severe
emotional distress (i.e., more than a reasonable person could be expected to endure). According to the
facts, the effect of the gas was temporary and none of (he pedestrians suffered any permanent damage.
Furthermore, the facts do not indicate that Parker suffered severe emotional distress. (D) is obviously
incorrect, as Gilda has in no way tortiously interfered with the privacy of Parker, who was on a public
sidewalk at the time. The tort of privacy includes the following four kinds of wrongs: (i) appropriation
by defendant of plaintiff’s picture or name for defendant's commercial advantage; (ii) intrusion by
defendant upon plaintiff’s affairs or seclusion; (iii) publication by defendant of facts placing plaintiff in
a false light; and (iv) public disclosures of private facts about the plaintiff by the defendant. The facts
do not suggest that Gilda committed any of these four kinds of wrongs.

5. (A) Even if Trucko can assert the defense of private necessity, he would still be liable for the
damage to Paget’s lawn. Trucko’s entrance onto Paget’s land constituted a prima facie case of trespass
to land. The following elements must be proved for trespass to land: (i) an act of physical invasion of
plaintiff’s real property by defendant; (ii) intent on defendant’s part to bring about a physical invasion
of the plaintiff’s real property; and (iii) causation. Intent to trespass is not required— intent to enter
onto the land is sufficient. Thus, even though Trucko did not intend to trespass onto Paget’s land, he did
intend to drive onto it to avoid hitting Crasher. Under the defense of private necessity, however, a
person may interfere with the property of another where it is reasonably and apparently necessary to
avoid threatened injury and where the threatened injury is substantially more serious than the invasion
that is undertaken to avert it. Where the act is solely to benefit any person or protect any property from
destruction or serious injury (rather than to benefit the public as a whole), the defense is qualified; i.e.,
the actor must pay for any injury he causes. Here, Trucko was acting to avoid serious injury. However,
he must pay for the damage to the lawn. (D) is therefore incorrect because negligence concepts are
irrelevant to an action based on trespass to land. (B) is incorrect because a private nuisance is a
substantial and unreasonable interference with another private individual’s use and enjoyment of his
land. Even if Trucko interfered with Paget’s use of his lawn, the interference was neither substantial nor
unreasonable. (C) is incorrect because there is no public nuisance in this case. A public nuisance
unreasonably interferes with the health, safety, or property rights of the community. Trucko's actions
affected one individual rather than the community.

6. (A) Missing Explanation

7. (C) Many students tend to confuse tortious assault with criminal assault. In order to be liable for
tortious assault, the defendant must act intending to cause a harmful or offensive contact with another
person or intend to cause an imminent apprehension of such a contact and the plaintiff must be placed

450121682.doc Page 36 of 57
in such imminent apprehension. Since Sammy was unaware that the gun was loaded, he was not placed
in imminent apprehension. Therefore, Calvin is not liable for assault and choice (C) is thus correct.

8. (D) An actor is subject to liability to another for false imprisonment if (a) he acts intending to
confine the other or a third person within boundaries fixed by the actor, and (b) his act directly or
indirectly results in such a confinement of the other. Restatement Section 37 provides, "If an act is done
with the intent to confine another, and such act is the legal cause of confinement to another, it is
immaterial whether the act directly or indirectly causes the confinement." Thus, one who institutes
criminal proceedings against another intends to cause an arrest which is the normal incident of such
proceedings. In such case, however, the actor is liable only if the confinement which the arrest involves
is a part of the greater offense of instituting such proceedings without probable cause and for a purpose
other than that for which the proceedings are provided. In the present example, the facts do not indicate
that Mr. and Mrs. Knight intended to have Dino arrested. They merely telephoned the police to
complain about the noise from his party. Therefore, since the police arrested Dino on their own accord,
Mr. and Mrs. Knight would not be liable for false imprisonment.

9. (A) In accordance with Restatement of Torts, 2d, Section 13, an actor is subject to liability to
another for battery if (a) he acts intending to cause a harmful or offensive contact with the person of the
other or a third person, and (b) a harmful or offensive contact with the person of the other directly or
indirectly results. Students should be aware that the defendant is liable not only for contacts which do
actual physical harm, but also for those relatively trivial ones which are merely offensive and insulting.
In the case presented herein, the Ames Board of Bar Examiners is vicariously liable for the intentional
tort (i.e., the unpermitted touching) committed by its employee. In general, the master is held liable for
any intentional tort committed by the servant where its purpose, however, misguided, is wholly or in
part to further the master's business.

10. (A) By process of elimination, choice (A) is the best answer. Here, choice (B) is wrong because
in order to recover for false imprisonment it is essential that the restraint be against the plaintiff's will.
If Jane agreed of her own free choice to surrender her freedom by accompanying Carol, then there was
no imprisonment. Choices (C) and (D) are incorrect because confinement may result by means other
than physical force. For example, confinement may be caused by physical barriers, by threats of
physical force, by duress, by asserted legal authority, etc. Consequently, alternative (A) is the best of
the given choices because the tort of false imprisonment protects the personal interest in freedom from
restraint of movement. Prosser notes that "imprisonment, while it seems originally to have meant stone
walls and iron bars, no longer signifies incarceration; the plaintiff may be imprisoned when his
movements are restrained in the open street, or in a traveling automobile, or when he is confined to an
entire city or is compelled to go along with the defendant." Thus, if Jane reasonably believed that she
would not be permitted to complete her bar examination unless she accompanied Carol, that would
constitute sufficient restraint upon the plaintiff's freedom of movement to be false imprisonment.

11. (C) A person is subject to liability for battery if he or she acts intending to cause a harmful or
offensive contact with the person of the other and an offensive or harmful contact directly or indirectly
results. An act which is not done with such an intention does not make the actor liable for mere
offensive contact. Since Keiki vomited involuntarily, she would lack the requisite intent to be liable for
tortious battery.

450121682.doc Page 37 of 57
12. (B) is the best answer. A person is entitled to defend another person from attack in the same
manner and under the same conditions as the person attacked would be entitled to defend himself.
Deadly force is permissible in self defense only to ward off an attack which itself threatens death or
serious bodily harm. Glitter is therefore privileged to threaten Cruiser with a knife—only to prevent
Cruiser from using deadly force—that which would inflict serious bodily injury—upon Flash. If
Cruiser's ramming of Flash's head into a car was itself application of deadly force (which it likely is)
Glitter was privileged to threaten Cruiser with deadly force to make him desist. If Cruiser's attack was
not deadly, Glitter's force in defense was excessive, and he is guilty of assault. (A) is incorrect. As
indicated above. Glitter was privileged to threaten to use deadly force if that amount of force was
reasonable under the circumstances—if Cruiser was threatening Flash with imminent serious bodily
injury. Thus, Glitter’s liability turns not merely on his use of deadly force but on whether Cruiser's use
of force against Flash was also deadly. Choice (B) identifies this controlling issue; choice (A) does not.
It is therefore incorrect. (C) is incorrect. Even though Cruiser was the original aggressor, Glitter would
not be privileged to threaten Cruiser with deadly force if Cruiser threatened only to inflict slight bodily
injury upon Flash. If Cruiser did not threaten serious bodily injury, the use of the threat of deadly force
by Glitter would not be reasonable in defense of Flash, just as if Flash were defending himself from a
non-serious attack, he would not be privileged to use deadly force. Since choice (C) erroneously
focuses on Cruiser's status as aggressor, (B) is a better answer. (D) is incorrect. Traditionally, defense
of others was only available where the person being defended was a member of the defendant's
immediate family. Modemly, Glitter is privileged to come to the aid of anyone threatened with serious
bodily injury, whether Glitter is related to the person or not. Thus, choice (D) also misidentifies the
controlling issue. Glitter's liability turns on the reasonableness of his use of deadly force, not his
relationship to Flash.

13. (A) is the best answer. Catherine cannot be held liable without fault. One possible theory is that
her use of the boat to purchase bread exceeded the consent given by Thomas, and thus constituted a
trespass to chattels or a conversion of the boat. However, the deviation from the main objective for
which Catherine borrowed the boat from Thomas was so minor as to not exceed the scope of the
consent which Thomas had given. Thomas did not expressly limit Catherine's use of the boat to driving
to the party and back. Thus, Catherine could not be liable for either of the intentional torts dealing with
damage to property, trespass to chattels or conversion. The other possible theory is negligence—she
could be liable if her manner of parking the boat near the grocery was negligent. Since choice (A)
mentions the only viable theory under which Thomas could prevail, it is the best answer.(B) is
incorrect. Catherine did not exercise dominion over the boat outside the scope of the consent given to
her by Thomas. Thus, as indicated above, she cannot be liable for an intentional tort such as conversion.
(C) is incorrect. Catherine did not exercise dominion over the boat outside the scope of the consent
given to her by Thomas, as indicated above. In addition, even if the minor deviation could be
considered a trespass to chattels, the damage to the boat was not sufficiently serious to justify forcing
her to pay for the entire value of the boat.(D) is incorrect. There is nothing in the facts to indicate that
Catherine used the boat for an unauthorized purpose.

14. (B) is the best answer. When Steve specified that Tad should stay inside the bay he was closely
defining the scope of authorized use. Since Tad deviated from the scope of the consent by crossing open
ocean that would pose significantly greater risks of damage to the boat, he will be liable if the boat is
damaged. Because the interference with Steve's ownership rights is relatively minor, Tad will be liable
in trespass to chattels rather than conversion. Damages available for trespass to chattels are those
actually suffered, such as the cost of repairing the water-damaged upholstery.(A) is not correct. Even

450121682.doc Page 38 of 57
though Tad was not at fault in causing the water damage to the upholstery, he is liable for the
intentional tort of trespass to chattels because he exceeded the scope of consent to use given by Steve.
Thus, he is liable for the actual damages suffered by Steve as a result of his intentional interference
with Steve's right to restrict use of the boat.(C) is incorrect. Tad is not liable for the value of the boat
merely because it was damaged while in his possession. Were Tad guilty of conversion, this would be a
proper measure of damages, since the owner of converted chattels need not accept their return.
However, the interference with ownership rights must be severe to constitute conversion, such as
wrongful acquisition, destruction, etc. A minor deviation in use is merely a trespass to the chattel, and
only actual damages are recoverable.(D) is incorrect. Tad did not exercise dominion and control over
the boat, which would constitute a conversion, merely by crossing one mile of open ocean. Tad had
Steve's consent to use the boat, and the minor deviation from Steve's instructions is not so inconsistent
with Steve's rights as owner to amount to the serious interference necessary for conversion. It is only a
trespass to chattel, and the greater damages recoverable under conversion are not available.

15. (C) Missing

16. (C) is the best answer. . Guy was privileged to trespass on Basil's property because of the private
necessity of saving himself from death or serious injury in the snowstorm. Because of this privilege,
Basil did not have the right to eject Guy into the storm and committed battery when he did. (A) is not
correct. Because of the life-threatening storm, Guy was privileged to be in Basil's cabin even after
Basil asked him to leave.(B) is not correct. Whether Basil's actions are reasonable or not is irrelevant in
light of the law's dictate that Basil had no right to eject Guy during the course of a severe storm as long
as his presence posed no threat of injury to Basil.(D) is not correct. Guy committed minimal damage to
Basil's cabin for which he is obligated to compensate Basil, but Guy showed no design to do further
damage to the cabin or to harm Basil. Moreover, Basil would not be privileged to use deadly force,
such as ejecting someone into a blizzard, merely to protect personal property.

17. (D) is the best answer. A person may invade the property rights of another where it is reasonably
necessary to avoid a substantially greater harm to himself. Thus, John was privileged by private
necessity to enter Ogre's land to facilitate his and Mary's escape from the forest fire, but he still must
compensate Ogre for any damage done to his land. Being privileged, and thus not liable for trespass,
(and not subject to forcible ejection by Ogre) John is still liable for any physical damage to the
property. (A) is not correct. The emergency provides the privilege to enter the property. However, the
privilege created by necessity does not include immunity from liability for damage done to Ogre's
property. Ogre, who was not responsible for the emergency and is innocent with regard to John's entry,
is entitled to recompense for any damage to his property in the entry. (B) is incorrect. While there might
conceivably be a rule exempting a privileged entrant from liability for damage so small as to be
negligible, the destruction of the sign is not so little that the law will not notice it. Ogre is entitled to
compensation for its destruction, as explained above. (C) is not the best choice. As discussed above,
John is not technically a trespasser, since he was privileged to enter onto Ogre's property to save
himself and Mary from the forest fire, under the doctrine of private necessity. John is liable for the sign
because he destroyed it, and his privilege does not extend to damaging or destroying the property upon
which he entered out of necessity.

18. (C) is the best answer. Ogre intended to place John and Mary in apprehension of an imminent
harmful touching, by shooting an arrow near them. This is sufficient intent to support liability for
assault. Generally, if a defendant intends to commit assault, battery, or false imprisonment (all original

450121682.doc Page 39 of 57
writs of trespass) and inadvertently commits an unintended one of the other torts, his intent will
“transfer” to support the tort appropriate for the result achieved. Since Ogre actually caused a harmful
contact. Ogre is liable for battery as to John, the victim. (A) is not correct. As was seen in connection
with the previous answer, John was technically not a trespasser. Because he was privileged by private
necessity to be on Ogre's land. Ogre was not privileged to use any degree of force to expel him. Ogre's
mistaken belief about John's status also provides no defense. (Even if John had been a trespasser. Ogre
would have been entitled to use only that force reasonably necessary to remove John from his land.
Shooting an arrow from concealment, with no prior demand that John leave (which John was doing
anyway) would never be considered reasonable force.) Ogre is liable for battery as explained above. (B)
is incorrect. Issues of superseding force, appropriate to a proximate cause analysis in negligence, are
not generally considered in intentional tort situations, especially where the defendant's intentional act is
a direct cause of the harmful result. Ogre intended to frighten John, an assault, and under the doctrine of
“transferred intent,” this mental state is sufficient to support liability for battery if the force used
actually results in a harmful contact. (D) is not the best choice. As indicated above, the fact that John
was or was not a trespasser (he was not) is not controlling under these circumstances. Ogre was not
privileged to shoot an arrow from concealment in order to expel supposed trespassers who were leaving
his land anyway. Such force greatly exceeded what was reasonably necessary to cause John to leave
Ogre's land. even under his mistaken assessment of the situation. By suggesting that John's status as a
trespasser would have resulted in a different legal conclusion, choice (D) is incorrect.

19. (D) is the best answer. Ogre’s action in shooting an arrow near Mary, intending to “frighten” her,
intentionally placed Mary, who must have seen the arrow pass right in front of her, in apprehension of
an immediate harmful or offensive touching. By definition, this is an assault. (A) is incorrect. Actual
physical contact is not an element of assault, which requires only that the plaintiff have been placed in
apprehension of an immediate harmful or offensive contact. (B) is incorrect. Marty made an
unauthorized entry on Ogre's land as a matter of private necessity to avoid injury from the forest fire.
The fact that Ogre mistakenly believed Mary was a trespasser does not relive him of liability for an
assault. (C) is incorrect. As has been seen in previous answers, Mary was not a trespasser, and was
privileged by private necessity to be on Ogre's private property. Thus, Ogre had no right to use any
force against Mary. By suggesting that Ogre's liability is based solely on his use of excessive force (i.e.,
more than reasonably necessary to expel an intruder), choice (A) is inaccurate.

20. (D) is the best answer. To be liable for false imprisonment, the clerks must have desired to
confine Franklin or known that there was an extremely high risk, equivalent to being substantially
certain, that he would be confined. If the clerks actually knew Franklin was still in the dressing room at
the time they locked the store, it was not only substantially certain, but inevitable, that Franklin would
be confined.(A) is not the best answer. The clerks should have known that a customer entering a
dressing room shortly before closing might still be there. Reasonable clerks would know that if they did
not check the dressing rooms, there was a risk someone would be left in the store. Some risk, however,
is not sufficient for false imprisonment; some risk is sufficient for negligence. This question asks the
student to address a false imprisonment cause of action.(B) is incorrect. Franklin need not suffer actual
damage to recover in false imprisonment. Awareness of the imprisonment is enough.(C) is not the best
answer. The fact that most patrons leave dressing rooms quickly is not relevant to what the clerks knew
about Franklin's presence when he was confined.

21. (C) is the best answer. Milton touched Franklin when he grabbed Franklin by the arm and
hustled him into the security offices. Milton will argue that he was using reasonable force in defense of

450121682.doc Page 40 of 57
his property. However, when Milton concluded that Franklin was trespassing, he bore the risk that he
was mistaken as to Franklin's right to be on the premises. Although Franklin was on the premises after
closing time, Franklin had entered the premises initially with Milton's consent during store hours and
ended up in the store after hours through no fault of his own. Franklin was privileged to be on Milton's
premises at that time and Milton will be liable for any unauthorized touching to him. (A) is not the best
answer. Even if Milton’s belief that he was defending his property was reasonable under the
circumstances, since Franklin was privileged to be on the premises, and Milton used force to attempt to
eject him, Milton will be liable for battery. (B) is not the best answer. Franklin did not exceed the scope
of the consent for him to be on the premises because he entered the store at a time it was open and was
left behind when the store closed earlier than usual through no fault of his own. (D) is incorrect. Milton
will be liable for a touching to Franklin whether or not the force that he used was unreasonable under
the circumstances.

22. (B) is the best answer. Milton would be privileged under the “Shopkeeper's Privilege” to confine
Franklin if Milton had a reasonable belief that Franklin had stolen merchandise and if Milton confined
Franklin for a reasonable time and in a reasonable manner to complete a reasonable investigation.
When Milton left for the banquet without telling Franklin that he could leave, Milton's investigation
and detention became unreasonable and Milton became liable for false imprisonment. (A) is incorrect.
Milton is privileged to detain a suspected thief, but only for a reasonable time to complete an
investigation. (C) is not the best answer. The absence of a means of escape is necessary for Franklin to
succeed in a cause of action for false imprisonment, but Franklin would lose even without a means of
escape if Milton was privileged to detain him. (D) is not the best answer. Once Milton informed
Franklin that he was calling the police, a threat of force was being used to confine Franklin. That
Milton later decided not to call the police is irrelevant.

23. (B) is the best answer. Self-defense is available if a reasonable person in the defendant's
circumstances would have believed that he was under attack, even if it is subsequently shown that no
actual harm was threatened. This concept has both a subjective and an objective component—the
defendant must honestly believe that he is threatened with harm, and it must be reasonable to so
believe. Here, choice (B) identifies these elements correctly. Note that by using the “if’ formulation,
choice (B) avoids the issue of whether Mitciv's belief actually was reasonable. (A) is incorrect. If
Mitciv honestly believed that Waitaminit was Rellik and was about to attack him, but a reasonable
person in the circumstances would not have concluded that, Mitciv's actions would not be privileged
and he would be liable to Waitaminit for battery.(C) is incorrect. Even though Mitciv is technically an
aggressor (having attacked Waitaminit first), the principle discussed in connection with choice (B)
operates to make Mitciv's “attack” privileged, if he honestly and reasonably believed that such action
was necessary to defend himself. Mitciv's reasonable mistake justifies the “attack” on Waitaminit.(D) is
incorrect. This is a variation on the theme of choice (A). Waitaminit did not “provoke” the attack by
Mitciv (except in the sense that Waitaminit's voice and words caused Mitciv to mistake him for Rellik),
but this is not controlling in the self-defense analysis. If Mitciv acted in good faith and reasonably, his
attack on Waitaminit is privileged.

24. (D) is the best answer. A battery occurs when the defendant touches any item intimately
connected to the plaintiffs person in a hostile or rude manner. An actual touching of the person is not
always required. LaFleur intended to touch the puck which Jim was holding in his hands at the time.
This is a sufficient connection to Jim's person to be a battery.(A) is incorrect. LaFleur touched an item
sufficiently intimately connected to Jim's person to be considered part of his person.(B) is incorrect. In

450121682.doc Page 41 of 57
working as a referee, Jim consented to incidental accidental touching but not to intentional attacks upon
his person.(C) is not the best answer. A battery requires an offensive touching of another, but the
touching need not always be a touching of the plaintiffs person. It can be a touching of an object
intimately connected to the plaintiffs person.25. (A) is the best answer. When LaFleur batted at the
puck in Jim's hands, he intended to commit a battery upon Jim. His intent to batter Jim transfers to
provide the intent in a cause of action for Charles against LaFleur for the harmful touching to his finger
by the flying puck.(B) is not correct. Charles' status as a trespasser on the premises does not bestow
upon others on the premises the right to batter Charles.(C) is incorrect. The intent to touch Jim
transfers to provide the intent to touch Charles.(D) is incorrect. LaFleur's expectations with regard to
the potential trajectory of the puck are irrelevant. When LaFleur chose to engage in conduct designed to
cause an unauthorized touching of another, he bore the risk that a touching of someone other than his
target would also occur.

25. (A) is the best answer. When LaFleur batted at the puck in Jim's hands, he intended to commit a
battery upon Jim. His intent to batter Jim transfers to provide the intent in a cause of action for Charles
against LaFleur for the harmful touching to his finger by the flying puck. (B) is not correct. Charles'
status as a trespasser on the premises does not bestow upon others on the premises the right to batter
Charles. (C) is incorrect. The intent to touch Jim transfers to provide the intent to touch Charles. (D) is
incorrect. LaFleur's expectations with regard to the potential trajectory of the puck are irrelevant. When
LaFleur chose to engage in conduct designed to cause an unauthorized touching of another, he bore the
risk that a touching of someone other than his target would also occur.

26. (B) In this situation Robinson would be liable for tortious assault if Reynolds feared that he
would hit him. Simply defined, assault is an act by the defendant creating a reasonable apprehension in
plaintiff of immediate harmful or offensive contact to plaintiff's person. Choice (A) is wrong because
Robinson is liable for the intentional tort of assault not negligence. Choice (C) is likewise incorrect
because it is not necessary to prove actual damages to sustain an action for assault. Note that choice (D)
is wrong because once apprehension has been intentionally created, it is no defense that the defendant
meant to play a joke or changed his mind.

27. (A) Here's a tricky Multistate example in which most students will narrow the correct answer
down to alternatives (A) or (C). Where the mental distress is caused by the defendant's conduct which
is not directed at the plaintiff, but at a third person, certain problems result. In order for the plaintiff to
recover in such cases, two requirements generally must be established: (1) the defendant knew of the
presence of the third person and (2) the plaintiff was a close relative of the person attacked. However,
with respect to the second requirement that recovery should be limited to near relatives, Prosser states
that although most cases allowing recovery have involved members of the immediate family, there are a
few which have not. However, in order for a third party non-family member to recover for infliction of
emotional distress, he or she must suffer bodily harm or injury. Since Haden suffered a heart attack
from witnessing the hatchet assault, choice (A) is correct.

28. (D) When a person has reasonable grounds to believe that he is being or is about to be attacked,
he may use such force as is reasonably necessary for protection against the potential injury. Remember
that the actor need only have a reasonable belief as to the other party's actions, i.e., apparent necessity,
not actual necessity, is sufficient. As a result, reasonable mistake as to the existence of the danger does
not vitiate the defense. Choice (D) is therefore correct because Hunter reasonably acted in self-defense

450121682.doc Page 42 of 57
when he shot at Caine. Caveat: If, in the course of reasonably defending himself, one accidentally
injures a bystander, he is nevertheless protected by the defense.

29. (C) Here's another difficult Multistate example where both alternatives (C) and (D) are arguably
correct. To be liable for assault, the defendant must have intended to interfere with the plaintiff's
personal integrity — which is to say that he must have intended to bring about an assault or a battery.
But the intent need not necessarily be to inflict physical injury, and it is enough that there is an intent to
arouse apprehension. Thus, Prosser points out that "it is an assault to fire a gun not aimed at the plaintiff
for the purpose of frightening him, or to point it at him when the defendant knows that it is unloaded,
and the plaintiff does not." Law of Torts, pg. 41. Therefore, Caine will be liable for assault if he
intended to frighten Hunter regardless of whether he actually intended to shoot him.

30. (B) One who by extreme and outrageous conduct intentionally or recklessly causes severe
emotional distress to another is subject to liability for such emotional distress, and if bodily harm
results, for such bodily harm as well. In the present case, Earhardt's conduct was extreme and
outrageous. Furthermore, Cass suffered sufficient emotional distress because the facts indicate that she
was panic-stricken. For bar exam purposes, emotional distress includes any of the following types of
mental suffering: fright, horror, grief, shame, humiliation, embarrassment, anger, chagrin,
disappointment, worry and nausea. Note that (D) is wrong because physical injury is not a required
element for intentional infliction of emotional distress.

31. (A) Under the majority view, since the law has always placed a higher value upon human safety
than upon mere rights in property, it is the accepted rule that there is no privilege to use any force
calculated to cause death or serious bodily injury to repel the threat to land or chattels, unless there is
also such a threat to the defendant's personal safety as to justify self-defense. Prosser points out that
"even the tradition that a man's house is his castle, and that one may kill in defense of his dwelling, has
given way in most jurisdictions to the view that such force is not justified unless the intrusion threatens
the personal safety of the occupants". Prosser, pg. 115.

32. (D) A possessor of land owes a duty to warn trespassers of known dangerous conditions which
are not obvious to the trespasser. In the present hypo, Munchkin was under no duty to warn trespasser-
Winslow of Defense's use of deadly force since Munchkin had ordered Defense to discontinue its
protection service. Consequently, Defense was not authorized to use deadly force under the
circumstances. Choice (C) is wrong because this question hinges on the issue of whether Munchkin
knew or had reason to know that Defense would be continuing to provide protection service for his
property. If he had reason to know, then Munchkin would have been under a duty to warn trespassers of
Defense's "armed response" policy. Note that choice (B) is wrong because it is irrelevant whether
Winslow knew that Defense provided an "armed response". On the contrary, the "key" issue is whether
Munchkin, as landowner, was aware of the danger presented. If so, then he would be liable for not
warning trespassers of the dangerous condition (namely, Defense's "armed response").

33. (C) One is subject to liability for trespass, irrespective of whether he causes harm to any legally
protected interest of the other, if he intentionally enters the land in possession of the other. Based upon
the given facts, Vic did not intentionally drive his vehicle onto Edna's property. Rather he lost control of
his vehicle while trying to negotiate a sharp curve in the highway. In this regard, Vic may be liable for
trespass because he was operating his car in a reckless manner. However, where a person enters the
land of another through negligence, recklessness or as a result of an abnormally dangerous activity, in

450121682.doc Page 43 of 57
order to be liable for trespass he must cause damage to the land. Therefore, choice (C) is a better
answer than (A) because even though Vic was reckless, he will not be liable unless he damaged Edna's
property. Note that choice (B) is not correct because Vic's entry was not intentional.

34. (C) Mistake is no defense for intentional trespass. Ott erroneously believed the channel was a
public waterway when, in fact, it was owned by Carr. Choice (C) is correct because it addresses Ott’s
intentional entry onto the channel. Note that choice (D) is wrong because the facts do suggest that an
easement by necessity was created since there was no conveyance of land.

35. (B) This same issue was tested on the February, 2003 Multistate Bar Examination. The issue
presented is whether a plaintiff may recover for infliction of emotional distress for injury caused to a
third person. The older decided cases held that recovery was allowed where (1) the plaintiff was present
and witnessed defendant's extreme and outrageous conduct and (2) where the plaintiff was a close
family member of the person attacked. With respect to non-family members, it is necessary of course
that they be present, but, in addition, their distress must result in bodily harm. As between answer
choices (A) and (B), note that (A) is wrong because Connie did in fact suffer bodily injury by having a
miscarriage. Note that choice (C) is not the best answer because breach of duty is an element in a
negligence action not in an infliction of mental distress case. Likewise, choice (D) is incorrect because
"foreseeability" applies in determining proximate cause for negligence. In emotional distress cases
where defendant's conduct is directed at a third person, it is only necessary that the plaintiff be present
at the time of the incident. Exam Tip: To be sure, this is an extremely difficult Multistate question.
Once again, when you are confronted by similarly worded answer choices, it is necessary to proceed by
process of elimination and not confuse the elements of one cause of action (such as negligence) with
another (as in this cause with infliction of emotional distress).

36. (D) Trespass to land is heavily tested on every bar exam. The two key distinctions deal with
intentional entries and negligent or reckless entries. If the defendant intentionally enters onto the
property of another, he or she is liable for trespass regardless of whether there is any damage to the
land. On the other hand, in negligent (or reckless) trespass situations, there must be damage to the land
in order to be held liable. In this problem Margo sued Scottie for trespass. The unanswered question is
whether this is an intentional or negligent trespass action. As a general rule on the MBE, you should
assume the action is intentional trespass unless the facts indicate otherwise. Thus, choice (D) is the best
answer. Note that Statement I is wrong because mistake is no defense for intentional trespass.
Statements II and III are not applicable because they address defenses to negligent entries. Therefore,
choice (D) is correct.

37. (D) Students should be aware that an actor is subject to liability to another for battery if he acts
intending to cause a harmful or offensive contact with the person of another. In the present
hypothetical, Granny did not act intending to cause a harmful or offensive contact, and therefore,
liability for battery would not attach.

38. (B) Note that an actor is subject to liability for battery if he acts intending to cause a harmful or
offensive contact with the person of another. Since the essence of the plaintiff's grievance consists in
the offense to the dignity involved in the unpermitted and intentional invasion of the inviolability of his
person and not in any physical harm done to his body, it is not necessary that the plaintiff's actual body
be disturbed.

450121682.doc Page 44 of 57
39. (C) According to Restatement of Torts 2d, Section 19, "a bodily contact is offensive if it offends
a reasonable sense of personal dignity." Comment (a) further states that "in order that a contact be
offensive to a reasonable sense of personal dignity it must be one which would offend the ordinary
person and as such one not unduly sensitive as to his personal dignity." Thus, the contact must be
unwarranted by the social usages prevalent at the time and place at which it is inflicted. Under the
circumstances presented here, Rider's conduct would be socially acceptable and therefore, he would not
be liable for any offensive touching or harmful contact.

40. (B) Under Section 46 Restatement of Torts 2d, one who by extreme and outrageous conduct
intentionally or recklessly causes severe emotional distress to another is subject to liability for such
emotional distress. Moreover, where such conduct is directed at a third person, the actor is subject to
liability if he intentionally or recklessly causes severe emotional distress to a member of such person's
immediate family who is present at the time.

41. (C) Where the extreme and outrageous conduct is directed at a third person, the cases thus far
decided have limited such liability to plaintiffs who were present at the time, as distinguished from
those who discover later what has occurred.

42. (D) An actor is subject to liability to another for battery if (a) he acts intending to cause a
harmful or offensive contact with the person of the other or a third person, or an imminent
apprehension of such a contact, and (b) an offensive contact with the person of the other directly or
indirectly results. Since Lester intended to douse Judge Cooch with water (from the sprinkling device),
he would be liable for a battery. In addition, by directing water over Judge Cooch's property, Lester
would be liable for trespass. Under Restatement of Torts, 2d, Section 158, "one is subject to liability to
another for trespass, irrespective of whether he thereby causes harm to any legally protected interest of
the other, if he intentionally (a) enters the land in the possession of the other, or causes a thing or a third
person to do so.

43. (A) Another popular testing area on the Multistate deals with situations involving an arrest
without a warrant As a general rule, either an officer or a private citizen may arrest without a warrant to
prevent a felony or breach of peace which is being committed, or reasonably appears about to be
committed, in his presence. Once the crime has been committed, however, the private person may still
arrest but his authority depends upon the fact of the crime, and he must take the full risk that none has
been committed. In the present case, the killing of the Riverdale police officers did not occur in
Nelson's presence. As a result, he must take the full risk for falsely arresting Jones. Therefore, by
pointing the pistol at Jones, Nelson would be liable for assault.

44. (B) A private person may arrest if a felony has in fact been committed, and she has reasonable
grounds to suspect the man whom she arrests has committed the crime. Here, the facts state that the
killings were committed by an individual who was a paraplegic and Caucasian. As a consequence,
Nelson did not have reasonable grounds to believe that Jones was the suspected felon since he was a
black man walking down the street. Note that (B) is a better answer than (A) because Nelson would
have been privileged to detain Jones had his belief been reasonable.

45. (D) Here's a classic Multistate example dealing with non-liability for "accidental entries on land.
In accordance with Restatement of Torts 2d, Section 166, "Except where the actor is engaged in an
abnormally dangerous activity, an unintentional and non-negligent entry on land in the possession of

450121682.doc Page 45 of 57
another, or causing a thing or third person to enter the land, does not subject the actor to liability to the
possessor, even though the entry causes harm to the possessor or to a thing or third person in whose
security the possessor has a legally protected interest."

46. (D) Paola will not recover in a suit for battery because Dixon’s contact did not constitute a
harmful or offensive contact. In order to establish a prima facie case for battery, the following elements
must be proved: (i) an act by the defendant that brings about harmful or offensive contact to the
plaintiff’s person; (ii) intent on the part of the defendant to bring about harmful or offensive contact to
the plaintiff’s person; and (iii) causation. Judged by this standard, Dixon's conduct in trying to keep
Paola from falling in a crowded bus would not be harmful or offensive. Contact is deemed “offensive”
if the plaintiff has not expressly or impliedly consented to it. Consent may be implied from custom,
conduct, or words, or by law. Under these facts the consent would be inferred as a matter of usage or
custom. A person is presumed to consent to die ordinary-contacts of daily-life, which would include
contact resulting from assistance to a fellow passenger in a crowded bus. (A) and (B) are incorrect.
Even if Dixon intended to put his arm around Paola’s waist or touched Paola without her permission,
the touching was not harmful or offensive and therefore Dixon cannot be deemed to have committed a
battery. (C) is incorrect. Even if Dixon s pu ting^ is arm around Paola's waist was not by accident,
Dixon would not be liable for battery because un er the circumstances the touching was not harmful or
offensive.

47. (C) Pauline cannot make out a case for battery because she did not experience an offensive
touching. To make out a prima facie case for battery, the plaintiff must show an intentional act by the
defendant that caused harmful or offensive contact to the plaintiff’s person. Here, nothing indicates that
Pauline was touched in any way; thus, (C) is correct. (A) is incorrect because injury is not an element of
battery; battery can be established even absent a showing of injury or damages. (B) is incorrect because
if the elements of battery were present, the physician-patient privilege would not protect Donald here
because Pauline did not impliedly consent to the taping. When a patient consents to a doctor’s
treatment, she impliedly consents to all necessary touching that goes along with the treatment.
However, when the doctor goes beyond the scope of the acts consented to and does something
substantially different, the defense of consent is no longer available. Here, Pauline consented to talk
with Donald, but that is substantially different from agreeing to be the subject of an experiment. Thus,
if taping Pauline were a battery, Donald could not rely on implied consent to relieve him of liability.
(D) is incorrect because a benevolent motive is not a defense to a battery. If the other elements of
battery were present, the fact that the battery occurred to foster medical research would not be a defense
—that would allow psychiatrists to physically strike their patients merely to study their reactions!

48. (D) Wilma will prevail because Kerry intentionally caused an offensive touching. To make out a
prima facie case for battery. Wilma need only show an act by Kerry that brings about harmful or
offensive contact, intent by Kerry to bring about the contact, and causation. These elements are present
here, and no defense is available (provocation is not a defense—insulting words do not give one the
privilege to strike another). Therefore, (D) is correct. (A) and (B) are incorrect because, as indicated
above, neither actual provocation nor reasonable provocation is a defense to battery. (C) is incorrect
because even spontaneous acts are volitional movements, and that is all that is required to meet the
“act** requirement. As long as the act is triggered by the conscious mind, it will he considered a
volitional movement.

450121682.doc Page 46 of 57
49. (D) Scott will most likely prevail because he did not intend to frighten Roman onto Tim’s
property. For Him to succeed in his trespass suit, he must show that Scott intended to bring about a
physical invasion of Tim’s property. Scott did not chase Roman onto Tim's yard, nor did Scott intend or
know with substantial certainty that Roman would enter onto Tim's yard as a result of Scott's actions.
(A) is incorrect because a landowner is not automatically privileged to chase or otherwise cause third
persons to enter onto another's land to prevent the commission of a tort against his property. While the
landowner may have a qualified defense if the trespass was reasonable and apparently necessary to
protect his property from destruction or serious injury, the interference with Tim’s properly here did not
result from necessity. (B) is incorrect because it is not necessary to establish a prima facie case for
trespass to land that the defendant personally came onto the land; e.g., trespass exists where the
defendant floods the plaintiff’s land, throw's rocks onto it. or chases third persons upon it. (C) is
incorrect because even though Roman made the decision to go over the fence, Scott could still be liable
tion of causing Roman to enter upon Tim's land.

50. (A) Missing

51. (D) Calib will prevail if it reasonably appeared that Adam was about to inflict serious bodily
harm to Ben. Deadly force in defense of others is acceptable if the other is being threatened with
serious bodily harm; choice (D) presents that situation. (A) is therefore incorrect. (C) is wrong because
the fact that Adam started the fight does not give Calib the right to use deadly force. (B) is wrong
because it is irrelevant whom Calib thought was the aggressor; if it was apparent that Adam was about
to inflict serious bodily harm on Ben, Calib would have the right to intervene.

52. (B) Bernadette is likely to prevail because Danny deliberately damaged her car. A minor child
will be held liable for the consequences of his intentional tortious conduct whether or not he is “aware”
of the consequences of that conduct. Hence. (A) and (D) are wrong. (C) is a misstatement of the law.
Danny may be liable even though he is under the care of his parents.

53. (A) If Duncan had the requisite intent for assault (creation of reasonable apprehension), that
intent is sufficient for battery if contact occurs as the result of Duncan’s actions. This is one
ramification of the doctrine of transferred intent. The transferred intent doctrine applies where the
defendant intends to commit a tort against one person but instead commits a different tort against that
person. The intent to commit the one tort is transferred to the other tort. Here, Duncan’s intent to
commit an assault would satisfy the intent requirement for battery. (B) is incorrect because mere
offensive contact will not suffice for battery if the requisite intent is absent. (C) is incorrect because of
the doctrine of transferred intent and the fact that Duncan set in motion the process that resulted in
Paine’s being struck. (D) is incorrect because Duncan had the requisite intent to commit an intentional
tort, making the negligence of a third party irrelevant.

54. (A) Digby will be liable to Mary for intentional infliction of emotional distress if his conduct is
judged to be extreme and outrageous. To establish a prima facie case for intentional infliction of
emotional distress, plaintiff must prove (i) an act by defendant amounting to extreme and outrageous
conduct, (ii) intent to cause severe emotional distress or recklessness as to the effect of defendant’s act,
(iii) causation, and (iv) damages, i.e., severe emotional distress. If Digby’s conduct is judged to be
extreme and outrageous, he is liable because the other elements of the tort are present: Given Digby’s
conduct and Mary’s response, Digby was at least reckless as to whether his conduct would cause severe
distress, and the facts indicate that it did cause such distress. Thus, Mary will prevail under the

450121682.doc Page 47 of 57
circumstances in choice (A). (B) is incorrect because a threat of physical injury is not required to
establish intentional infliction of distress. Nor would threat of physical injury be sufficient to create
liability for negligent infliction of emotional distress—that tort requires also that some physical injury
result, which is not the case here. (C) is wrong because Digby could be liable even if he did not intend
for Mary to suffer emotional distress, as long as he was reckless as to the effect of his conduct. (D) is
incorrect because the fact that Mary owed the money would not excuse Digby’s conduct if it was
extreme and outrageous.

55. (B) Cisneros will win because LeBeau’s actions constituted an assault. A prima facie case for
assault consists of: (i) an act by the defendant creating a reasonable apprehension in the plaintiff of
immediate harmful or offensive contact; (ii) intent on the part of the defendant to bring about that
apprehension; and (iii) causation. Here, if Cisneros reasonably believed that LeBeau’s boat was about
to hit his boat (and thus cause a harmful or offensive contact with him), there is a basis for assault
because the other elements (intent on the part of LeBeau and causation) were present. The only issue is
whether the belief was reasonable. In determining whether the apprehension was reasonable, the courts
usually apply a reasonable person test. Here, a reasonable person certainly could believe that LeBeau’s
actions constituted a threat of an immediate harmful contact (e.g., a crash). Thus, there would be a basis
for assault. (A) is wrong because unless Cisneros’s initial actions were such as to justify LeBeau's
acting in self-defense, it is immaterial that Cisneros made insults provoking LeBeau’s tortious conduct.
Self-defense is permitted when a person reasonably believes he is being or is about to be attacked.
Here, it is clear that Cisneros's conduct would not justify LeBeau’s actions in self-defense. Cisneros
made no threat to LeBeau. Words alone (in the heated conversation between two parties) do not cause
reasonable apprehension of harmful contact, and there is no showing that any acts indicated harm. Also,
self-defense would not apply because LeBeau’s conduct was clearly not defensive; he repeatedly came
after Cisneros despite Cisneros’s attempts to leave. (C) is wrong because damages are recoverable for
assault without the plaintiff's suffering actual physical injury. Assault is the causing of apprehension of
contact; it does not require actual contact. (D) is wrong because Cisneros does not need to rely on an
intentional infliction of emotional distress to recover. He can prevail on an assault claim even if he did
not suffer severe emotional distress.

56. (A) Wanda will probably prevail on a claim for intentional infliction of emotional distress
because Morlock’s conduct was sufficiently extreme and outrageous and the other elements of the tort
are present. Intentional infliction of emotional distress requires: (i) an act by defendant amounting to
extreme and outrageous conduct; (ii) intent to cause severe emotional distress or recklessness as to the
effect of defendant’s conduct; (iii) causation; and (iv) damages. “Outrageous conduct” is extreme
conduct that transcends all bounds of decency. Morlock’s use of the bullhorn to amplify the recording
and his insults against Wanda for the benefit of her guests would probably qualify as extreme and
outrageous conduct, particularly since there is no evidence that he had previously tried to resolve the
problem with Wanda in a more civilized manner. Morlock had the requisite intent (either he intended to
cause emotional distress, as shown by use of the bullhorn, or he was reckless as to its effect), there was
causation, and Wanda suffered damages (i.e., she was severely distressed) as a result of Morlock’s
actions. (B) is wrong because pecuniary harm is not required for purposes of this tort—all that is
required is severe emotional distress. (C) is wrong because, in contrast to negligent infliction of
distress, intentional infliction of distress does not require proof of physical harm to recover. (D) is
wrong because the fact that the barking constituted a nuisance would not be a defense to conduct
amounting to intentional infliction of distress; abatement of a private nuisance by self-help must be
preceded by notice to the other party and must be conducted in a reasonable manner.

450121682.doc Page 48 of 57
57. (B) If Wanda knew the door would strike the bullhorn, she could be liable to Morlock for battery.
Battery requires: (i) an act by defendant that causes a harmful or offensive contact to plaintiff’s person;
(ii) intent to cause the harmful or offensive contact; and (iii) causation. Here, there was a harmful
contact caused by Wanda. The only consideration is whether Wanda had the requisite intent. If a person
knows with substantial certainty the consequences of her action, she has the intent necessary for this
type of tort. Thus, if Wanda knew that the door was substantially certain to hit the bullhorn Morlock
was holding, Wanda had the intent necessary for battery. (For purposes of battery, anything connected
to or being held by the plaintiff is part of plaintiff’s person.) (A) is wrong because if Wanda intended to
cause a harmful contact (a battery), she is liable for all the consequences of her actions, whether she
intended them or not. A defendant need not foresee the extent of the injuries caused by her intentional
act to be held liable for them. (C) is wrong because this is not a case of self-defense. Self-defense is
appropriate when a person reasonably believes that she is being or is about to be attacked. Nothing in
the facts shows any basis for Wanda to believe that Morlock was going to harm her. Thus, self-defense
is not appropriate here. (D) is wrong because it does not provide Wanda with a defense. Morlock’s
conduct angered Wanda and may have triggered her actions, but since Morlock's conduct was not
sufficient to allow Wanda to act in self-defense, Wanda’s use of force here is not excused.

58. (D) is the best answer. Kendall threw the ball at Kim deliberately. Kendall's mental illness is not
a defense to the intentional tort of battery unless the illness is such that it prevents Kendall from
forming the intent to touch. Even if Kendall is unable to form the intent to harm or understand that the
ball may cause harm, he is liable for battery if he can form the intent to touch. (A) is incorrect. There is
no rule of law that mentally ill adults are conclusively presumed incapable of forming the intent to
injure another. (B) is similarly incorrect. There is no rule of law that mentally ill adults are conclusively
presumed incapable of forming the intent to touch. (C) is incorrect. The reasonable person standard is
applicable to mentally ill adults in a negligence action, but Kim is suing Kendall for battery here so this
test is irrelevant.

59. (A) is the best choice. An assault is a volitional act, done with the requisite intent, which causes
plaintiff to experience a reasonable apprehension of an immediate, harmful or offensive contact. If the
defendant threatens harm in the future, or is discemably unable to carry out the threat, there is no
assault. Here, Dortmunder threated that he would bum down Peltier's home, with Peltier inside,
manifestly a harmful physical contact, but the threat was to do so at some unspecified time in the
future. Dortmunder is therefore not liable for the intentional tort of assault. (B) is not the best choice.
An otherwise actionable threat may be rendered harmless by accompanying words indicating that the
act is symbolic or in jest (e.g., “If it weren't such bad manners, I'd slap your face”). However, the fact
that the defendant purported to qualify his threat in such a way that the plaintiff could ostensibly avoid
harm by compliance does not relieve the defendant of liability (unless the defendant was privileged to
impose the condition). Here, Dortmunder has no right to insist that his graffiti be left undisturbed on
Peltier's fence, and thus the purported qualification—that Peltier can avoid harm by leaving the fence
unpainted—is of no legal effect. If not for the lack of immediacy discussed above, Dortmunder would
be liable for assault. (C) is inaccurate. Intent to cause apprehension of a harmful physical contact (or
intent to cause such a contact) is an element of assault. In the present question, die element of
immediate harm is missing, and thus no liability for assault can be found even though other elements of
the tort are present. (D) is inaccurate. A threat of an immediate, harmful or offensive contact need not
be considered extreme or outrageous conduct in order to be actionable as an assault. Threatened contact
is harmful if it would inflict pain, injury, disfigurement or impairment of any bodily organ. Such
contact is offensive if a reasonable person of ordinary sensibilities would reagard it as offensive. In

450121682.doc Page 49 of 57
addition, even if Dortmunder's conduct were regarded as extreme and outrageous, the lack of an
immediate threat of harm precludes liability for assault.

60. (D) is the best choice. Intentional infliction of emotional distress is a volitional act, done with the
requisite intent, which amounts to extreme and outrageous conduct and which causes the plaintiff to
suffer severe emotional distress. Threatening to bum down the plaintiff's house while the plaintiff is
inside as retaliation for painting over graffiti is clearly extreme and outrageous conduct. This is
actionable if the defendant intended to cause severe emotional distress or recklessly disregarded the
high probability that it would occur. Under these circumstances, Dortmunder was acting intentionally
or at least recklessly, so if Peltier suffered severe emotional distress, Dortmunder is liable for the
charged tort(A) is inaccurate. Threatening immediate harm is an element of assault, not of infliction of
emotional distress. Even a threat of remote (in time) harm, if extreme and outrageous, is actionable if
the other elements of infliction of emotional distress are present. Choice (D) supplies the one element
not provided in the fact pattern, and thus is the correct answer.(B) is wrong. Where intentional
infliction of emotional distress is charged, the extreme and outrageous nature of the defendant's conduct
is held to make it unlikely that the plaintiff will fabricate his injuries. Thus, there is no requirement that
the plaintiff manifest physical symptoms or actually suffer physical harm. (Contract negligent infliction
of emotional distress, as to which most courts do require physical symtoms in order to permit recovery
for the mental harm.)(C) is incorrect. The defendant's intent to cause apprehension of a harmful
physical contact is an element of assault, not of infliction of emotional distress. For the latter tort, the
defendant must intend to cause severe emotional distress or recklessly disregard the very high
probability that such distress will occur. Dortmunder’s threat is evidence that he possessed one of these
mental states, but the nature of the threat is otherwise not material to an analysis of liability.

61. (B) is the best answer. Although the sludge pit appears to be a nuisance, the injury which has
occurred here is not an interference with the rights of the public in general as is necessary for a public
nuisance or an interference with someone's use and enjoyment of their land as is necessary for a private
nuisance. Jennifer’s daughter was injured when she trespassed on Phillip's land. Normally, land
occupiers do not owe trespassers any duty of care. However, if Phillip knew or had reason to know
children were likely to trespass, knew or had reason to know that there were conditions on the premises
dangerous to children, if the children because of their age were unable to recognize or appreciate the
danger, and the burden of correcting the danger was small in relation to the risk to the children, Phillip
would owe the trespassing children a duty of reasonable care. Since choice (B) states two of these
requirements, and the other alternatives are more clearly incorrect, (B) is thus the best answer.(A) is not
correct. Phillip need not actually know of Jennifer's daughter's presence. He need only know or have
reason to know that children are likely to trespass.(C) is incorrect. This is a distractor based upon the
label which is sometimes attached to the trespassing child problem—“attractive nuisance.” It is not
necessary in order for the special duty to child trespassers to arise that the injuring condition “attract”
the child onto the property. Thus, so long as the conditions discussed in connection with choice (B) are
met, the landowner will be liable even if the child came onto the land for a reason entirely unrelated to
the presence of the dangerous condition.(D) is incorrect.It is not necessary for the duty to arise that the
condition be a nuisance or a nuisance per se. Again, the “attractive nuisance” label is a misnomer based
upon early analysis which is no longer viable.

62. (A) is the best answer. To be liable for intentional infliction of emotional distress the defendant's
conduct must be extreme and outrageous, done with intent to cause or in reckless disregard of causing
the plaintiff severe emotional distress, and severe emotional distress must actually result. Proximate

450121682.doc Page 50 of 57
cause is not a concept ordinarily applied to intentional torts; if the defendant takes action to effect that
injury to plaintiff, the fact that the process involves an innocent third party in conveying the
information which causes the emotional distress is not regarded as having any ameliorating impact on
the issue of liability. Even if this were viewed as a negligent infliction of emotional distress problem,
and the son were to be judged negligent in abruptly showing the mirror and window to Triptolemus,
such negligence is clearly a foreseeable intervening force which would not relieve Barbara of liability.
Since Barbara intended that Triptolemus be injured by discovering the mirror and window, and this
desired result was achieved, a negligence analysis seems inappropriate even though the son's actions
formed an intervening step in the process. In any case, Barbara is clearly liable for inflicting emotional
distress on Triptolemus.(B) is incorrect. Assuming that the son's actions were negligent, such
negligence would be analyzed under causation principles of foreseeability, not as contributory
negligence. Contributory negligence has no applicability here, since Triptolemus is the plaintiff and did
nothing but suffer injury. Since choice (B) reaches the correct result for inaccurate reasons, it is not the
best answer.(C) is incorrect. As indicated above, the son's conduct, even if negligent, is either
immaterial under an intentional tort analysis, or is foreseeable and thus not superseding under a
negligence analysis.(D) is not correct. The son's actions were part of the causal chain which led to
Triptolemus' injury. As already seen, this has no legal effect upon Barbara's liability. Her actions were
also a cause in fact and the proximate cause of the injury.

63. (A) is the best answer. A defendant may be liable for intentional infliction of emotional distress
when he engages in extreme and outrageous conduct toward one person, intending to inflict emotional
distress upon a family member of the victim whom the defendant knows is present. Whether viewed as
a limited application of the “transferred intent” doctrine or of the principle that the defendant must be
substantially certain that a family member witnessing his attack upon the victim will suffer emotional
distress, it is necessary that the defendant be aware of the family member's presence. Since Waiter was
unaware of Mother's presence in the restaurant, he cannot have had the requisite intent as to her. (B) is
incorrect. The bystander need not herself be close enough to the victim to be threatened by physical
harm from the defendant's attack when the theory of recovery is intentional infliction of emotional
distress. The “zone of danger” requirement to which choice (B) alludes is the requirement in the
majority of jurisdictions for recovery under negligent infliction of emotional distress. (C) is incorrect.
In addition to causing severe emotional distress, the tort requires that the tortfeasor intend to cause or
act in reckless disregard of causing severe emotional distress as to the plaintiff. In regard to a bystander
plaintiff this intent is shown by the close relationship between the bystander and the victim and the
knowledge of the wrongdoer that the bystander is present, the latter element not being shown by the
facts in this scenario. (D) is not correct. Waiter's conduct being extreme and outrageous alone is
insufficient to prove intentional infliction of emotional distress to a bystander. Waiter must have
intended that Mother, the bystander, suffer emotional distress. If Waiter did not know that Mother was
present, he cannot have had such an intent.

64. (C) is the best choice. The key to analysis of this problem is the fact that Portsmouth suffered no
physical injury but clearly suffered mental distress. “Pure” mental distress (unaccompanied by physical
injury) is actionable as an intentional tort; most states deny recovery for negligent infliction of mental
distress if there are no physical symptoms of harm. A defendant is liable for intentional infliction of
mental distress if his extreme and outrageous conduct, intended to cause distress or in reckless
disregard of whether such distress might occur, causes the plaintiff to suffer severe mental distress. A
defendant is held to “intend” the consequences of his act if he is substantially certain that they will
follow from the act. Thus, if Dernier was substantially certain that moving the gangplank would result

450121682.doc Page 51 of 57
in Portsmouth's dunking. Dernier is liable for the humiliation which Portsmouth suffered.(A) is wrong.
So long as the defendant is acting intentionally or recklessly, the courts do not require that there be
actual physical harm accompanying a claim of infliction of mental distress. The wrongfulness of the
defendant's mental state, and the extreme and outrageous nature of his conduct make it likely that the
plaintiffs mental injuries are not fabricated.(B) is incorrect. A defendant is held to intend consequences
which he is substantially certain will follow from his acts, even if he does not subjectively “desire” that
they will happen. Thus, if Dernier was substantially certain that Portsmouth would fall into the water,
the fact that Dernier believed that this was a practical joke (and thus was not wrongful conduct) would
not relieve him of liability for the intentional tort. In addition, even with the mental state described in
choice (B), Dernier was still reckless in disregarding the likelihood that Portsmouth would suffer severe
mental distress from the dunking, and such recklessness is sufficient to support a judgment in
Portsmouth's favor. (D) is inaccurate. As indicated above, most courts will not impose liability for
negligent infliction of mental distress unless it is accompanied by some symptoms of physical harm.
Since the fact pattern here was careful to exclude phyiscal harm to Portsmouth, negligence was
eliminated as a basis for Dernier's liability.

65. (C) is the best answer. Private necessity permits a defendant to violate the property rights of
another in order to avoid a substantially greater injury to himself. The defendant's judgment in this
regard need not be certain; if a reasonable person would believe that the action taken was necessary to
avoid the requisite harm, the defendant is privileged even if he made a honest mistake in that regard.
Gerd is not liable for trespass if he had reasonable grounds to believe that he was in physical danger (or
even if his donkey was in danger) from the mountain lion. (A) is incorrect. As indicated above, Gerd is
within the privilege of private necessity if his belief in the justifying danger of the mountain lion was
reasonable, even if actually mistaken. Thus, the fact that, unknown to Gerd, the lion left the vicinity and
actually presented no danger is immaterial to his liability for trespass. Naem will not prevail. (B) is also
incorrect. Trespass does not require that the defendant know the land belongs to another, or even he
have a reasonable suspicion. In addition, regardless of the private or public ownership of the land, Gerd
would be privileged to enter if he reasonably concluded entry was necessary to prevent personal danger
or serious property loss.(D) is incorrect. If the perceived danger from the mountain lion was
reasonable, Gerd was privileged to enter the land to prevent the personal injury or the loss of his
donkey regardless of the state of its ownership. The gate being posted for no trespassing is again,
immaterial to the privilege analysis.

66. (C) is the best answer. Gerd was privileged to enter Naem's land with his donkey to prevent
personal injury or the loss of the donkey to the mountain lion. Because of this privilege, Gerd was not
committing a trespass and Naem was not entitled to knowingly subject the donkey to almost certain
destruction by locking it out of his property. Naem's actions amounted to a conversion (by destruction)
of the donkey.(A) is not correct. Private necessity protects loss or damage to personal property as well
as personal injury. Because the mountain lion threatened to destroy Gerd's donkey, Gerd would have
been entitled to enter Naem's land if only to protect the donkey, whose loss would be substantially
greater than the technical violation of Naem's property right.(B) is incorrect. As discussed in
connection with the previous question, Gerd was privileged by private necessity to be on Naem's land,
even without Naem's permission. Naem had no right to eject Gerd or Gerd's personal property. (D) is
not correct. Choice (D) goes too far. Naem had to suffer the invasion of his land under the doctrine of
private necessity. Naem had no affirmative duty to protect Gerd or Gerd's personal property. Naem is
liable for loss of the donkey because he caused its destruction without legal right.

450121682.doc Page 52 of 57
67. (D) The "key" to this question is carefully reading the interrogatory or "stem" which asks the
examinee to determine the cause of action that would afford Josephine her maximum recovery. Since
the cat is only worth $50, that would be the amount of recovery for conversion. In a conversion action,
the plaintiff is entitled to recover the full value of the chattel which in this case is $50. Likewise, choice
(C) is wrong because trespass to chattels allows recovery for the diminished value of the chattel which
would be an amount less than $50. Clearly, choice (D) is correct because the plaintiff would be able to
recover for her emotional distress which undoubtedly would be in excess of $50.

68. (A) In accordance with Restatement of Torts, 2d, Section 158, "One is subject to liability to
another for trespass, irrespective of whether he thereby causes harm to any legally protected interest of
the other, if he intentionally (a) enters the land in the possession of the other, or causes a thing or a third
person to do so, or (b) remains on the land, or (c) fails to remove from the land a thing which he is
under a duty to move." Students should also be cognizant that one may also be liable for trespass for
intrusions resulting from reckless or negligent conduct and abnormally dangerous activities. In this
regard, Restatement of Torts, 2d, Section 165, provides that "One who recklessly or negligently, or as a
result of an abnormally dangerous activity, enters land in the possession of another or causes a thing or
third person so to enter is subject to liability (for trespass)." Note that choice (B) is wrong because the
facts do not indicate that Marty either intentionally or negligently caused the ball to enter onto Dickie's
property.

69. (D) Students should be aware that no battery occurred. In accordance with Restatement of Torts,
2d, Section 18, "an actor is subject to liability to another for battery if (a) he acts intending to cause a
harmful or offensive contact with the person of the other and (b) an offensive contact with the person of
the other directly or indirectly results." It is important to point out, however, that an act which is not
done with the intention stated in subsection (b) does not make the actor liable to the other for a mere
offensive contact with the other's person. Thus, Marty, not intending to strike Dickie with the golf ball,
would not be held liable for battery. Similarly, Marty would not be liable for assault since he did not
intend to cause a harmful or offensive contact to Dickie's person, nor did he intend to put Dickie in
imminent apprehension of such conduct.

70. (D) In light of the preceding explanation, Marty would not be subject to tort liability to either.

450121682.doc Page 53 of 57
Answer key Questions Source Review
[2009-10] Multistate
1. C 45. D Testing
2. D 46. D 1. BLUE Book-
3. B 47. C PMBR Question
4. B 48. D Multistate 34 – page
5. A 49. D Workbook 269
6. A 50. A Volume 2 7. RED
7. C 51. D – Question PMBR
8. D 52. B 1 – page 1 Multistate
9. A 53. A 2. BLUE Workbook
10. A 54. A PMBR Volume 1-
11. C 55. B Multistate Question
12. B 56. A Workbook 81 – page
13. A 57. B Volume 2 89
14. B 58. D – Question 8. RED
15. C 59. A 2 – page 1 PMBR
16. C 60. D 3. BLUE Multistate
17. D 61. B PMBR Workbook
18. C 62. A Multistate Volume 1-
19. D 63. A Workbook Question
20. D 64. C Volume 2 189 – page
21. C 65. C – Question 115
22. B 66. C 3 – page 1 9. RED
23. B 67. D 4. GREEN PMBR
24. D 68. A BARBRI Multistate
25. A 69. D Bar Workbook
26. B 70. D Review Volume 1-
27. A Multistate Question
28. D Testing 190 – page
29. C Book- 116
30. B Question 10. RED
31. A 15 – page PMBR
32. D 263 Multistate
33. C 5. GREEN Workbook
34. C BARBRI Volume 1-
35. B Bar Question
36. D Review 191 – page
37. D Multistate 116
38. B Testing 11. RED
39. C Book- PMBR
40. B Question Multistate
41. C 31– page Workbook
42. D 268 Volume 1-
43. A 6. GREEN Question
44. B BARBRI
Bar

450121682.doc Page 54 of 57
114 – page 18. BLUE 19 – page Volume 1-
98 PMBR 5 Question
12. BLUE Multistate 24. BLUE 131 – page
PMBR Workbook PMBR 101
Multistate Volume 2 Multistate 30. RED
Workbook – Question Workbook PMBR
Volume 2 12 – page Volume 2 Multistate
– Question 3 – Question Workbook
5 – page 2 19. BLUE 20 – page Volume 1-
13. BLUE PMBR 5 Question
PMBR Multistate 25. BLUE 132 – page
Multistate Workbook PMBR 101
Workbook Volume 2 Multistate 31. RED
Volume 2 – Question Workbook PMBR
– Question 13 – page Volume 2 Multistate
6 – page 2 3 – Question Workbook
14. BLUE 20. BLUE 21 – page Volume 1-
PMBR PMBR 5 Question
Multistate Multistate 26. RED 144 – page
Workbook Workbook PMBR 104
Volume 2 Volume 2 Multistate 32. RED
– Question – Question Workbook PMBR
7 – page 2 16 – page Volume 1- Multistate
15. BLUE 4 Question Workbook
PMBR 21. BLUE 127 – page Volume 1-
Multistate PMBR 101 Question
Workbook Multistate 27. RED 145 – page
Volume 2 Workbook PMBR 104
– Question Volume 2 Multistate 33. RED
1 – page 2 – Question Workbook PMBR
16. BLUE 17 – page Volume 1- Multistate
PMBR 4 Question Workbook
Multistate 22. BLUE 128 – page Volume 1-
Workbook PMBR 101 Question
Volume 2 Multistate 28. RED 43 – page
– Question Workbook PMBR 80
10 – page Volume 2 Multistate 34. RED
3 – Question Workbook PMBR
17. BLUE 18 – page Volume 1- Multistate
PMBR 4 Question Workbook
Multistate 23. BLUE 130 – page Volume 1-
Workbook PMBR 101 Question
Volume 2 Multistate 29. RED 44 – page
– Question Workbook PMBR 80
11 – page Volume 2 Multistate 35. RED
3 – Question Workbook PMBR

450121682.doc Page 55 of 57
Multistate Workbook 38 – page Multistate
Workbook Volume 1- 270 Testing
Volume 1- Question 47. GREEN Book-
Question 27 – page BARBRI Question
62 – page 76 Bar 57 – page
84 Review 277
36. RED 42. RED Multistate 52. GREEN
PMBR PMBR Testing BARBRI
Multistate Multistate Book- Bar
Workbook Workbook Question Review
Volume 1- Volume 1- 40 – page Multistate
Question 5 Question 271 Testing
– page 71 28 – page 48. GREEN Book-
37. RED 76 BARBRI Question
PMBR 43. RED Bar 70 – page
Multistate PMBR Review 281
Workbook Multistate Multistate 53. GREEN
Volume 1- Workbook Testing BARBRI
Question 8 Volume 1- Book- Bar
– page 72 Question Question Review
38. RED 33 – page 44 – page Multistate
PMBR 78 273 Testing
Multistate 44. RED 49. GREEN Book-
Workbook PMBR BARBRI Question
Volume 1- Multistate Bar 77 – page
Question 9 Workbook Review 284
– page 72 Volume 1- Multistate 54. GREEN
39. RED Question Testing BARBRI
PMBR 34 – page Book- Bar
Multistate 78 Question Review
Workbook 45. RED 51 – page Multistate
Volume 1- PMBR 275 Testing
Question Multistate 50. GREEN Book-
12 – page Workbook BARBRI Question 8
73 Volume 1- Bar – page 591
40. RED Question Review 55. GREEN
PMBR 30 – page Multistate BARBRI
Multistate 77 Testing Bar
Workbook 46. GREEN Book- Review
Volume 1- BARBRI Question Multistate
Question Bar 52 – page Testing
26 – page Review 275 Book-
76 Multistate 51. GREEN Question
41. RED Testing BARBRI 16 – page
PMBR Book- Bar 594
Multistate Question Review

450121682.doc Page 56 of 57
56. GREEN Multistate 67. RED
BARBRI Workbook PMBR
Bar Volume 2 Multistate
Review – Question Workbook
Multistate 27 – page Volume 1-
Testing 6 Question 4
Book- 62. BLUE – page 71
Question PMBR 68. RED
17 – page Multistate PMBR
595 Workbook Multistate
57. GREEN Volume 2 Workbook
BARBRI – Question Volume 1-
Bar 97 – page Question
Review 24 14 – page
Multistate 63. BLUE 74
Testing PMBR 69. RED
Book- Multistate PMBR
Question Workbook Multistate
18 – page Volume 2 Workbook
595 – Question Volume 1-
58. BLUE 54 – page Question
PMBR 13 15 – page
Multistate 64. BLUE 74
Workbook PMBR 70. RED
Volume 2 Multistate PMBR
– Question Workbook Multistate
24 – page Volume 2 Workbook
6 – Question Volume 1-
59. BLUE 4 – page 1 Question
PMBR 65. BLUE 16 – page
Multistate PMBR 74
Workbook Multistate
Volume 2 Workbook
– Question Volume 2
25 – page – Question
6 14 – page
60. BLUE 3
PMBR 66. BLUE
Multistate PMBR
Workbook Multistate
Volume 2 Workbook
– Question Volume 2
26 – page – Question
6 15 – page
61. BLUE 3
PMBR

450121682.doc Page 57 of 57

Você também pode gostar